You are on page 1of 116

Classifications of Insulin: ------- Rapid acting insulin, short acting ins ulin, intermediate acting

insulin, long acting insulin, combination therapy (premixed). Rapid acting insulins: clear 1. aspart (Novolog) Onset ------- 10-20 minutes. Peak ------- 1-3 hours Duration-----3-5 hours Administration and timing ---- SubQ: 5-10 minutes before meal Compatibility: Can give with NPH; draw aspart up first, give immediately 2. lispro (Humalog)rapid acting analog of regular insulin Onset ------- 5-15 minutes. Peak ------- 1-1.5 hours Duration-----3-4 hours Administration and timing ---- SubQ: 5-10 minutes before meal Compatibility: Can give with NPH; draw lispro up first, give immediately 3. glulisine (Apidra) Onset -------15-30 minutes. Peak ------- 1 hour Duration-----3-4 hours Administration and timing ---- SubQ: 15 minutes before meal Compatibility: Can give with NPH; draw glulisine up first, give immediately Short acting insulin: (Regular) clear 1. Humulin R 2. Novolin R Onset ------- 30-60 minutes. Peak ------- 1-5 hours Duration-----6-10 hours Administration and timing ---- SubQ: 30-60 minutes before meal; IV Compatibility: Can mix with NPH, sterile water, normal saline; DO NOT MIX WITH GLARGINE

Intermediate acting Isophane susp a. NPH b. Humulin N

Onset ------- 1-2 hours Peak ------- 6-14 hours Duration-----16-24 hours Administration and timing ---- SubQ: Mix (Cloudy) Compatibility: Can mix with aspart, lispro, reg; DO NOT MIX WITH GLARGINE Long-Acting--Clear Detemir (Levemir) Onset ------- Gradual Peak ------- 6-8 hours Duration-----to 24 hours Administration and timing ---- SubQ: 1/day or 2/day Compatibility: DO NOT mix with any other insulin glargine Lantus

Onset ------- 1.1 hour Peak ------- No Peak Duration-----to 24 hours Administration and timing ---- SubQ: 1/day, same time each day Compatibility: DO NOT mix with any other insulin Combination Therapy Insulins (Premixed): ------- NPH/regular 70/30 (Humulin 70/30, Novolin 70/30, ReliOn 70/30), NPH/regular 50/50 (Humulin 50/50), lispro protamine/lispro 75/25 (Humalog Mix 75/25), lispro protamine/lispro 50/50 (Humalog mix 50/50), aspart protamine/aspart 70/30 (NovoLog 70/30). Type of Insulin/Frequency of Once a day regimen dose: ------- Intermediate (NPH) at bedtime, or long acting (glargine Lantus, determir Levemir) in am or at bedtime. Twice a day regimen dose? ------- Split mixed dose. Type of Insulin/Frequency of Twice a Day Regimen dose: ------- NPH and regular or rapid, before breakfast and at dinner. Three times a day regimen dose: ------- Combination of mixed and single dose.

Type of Insulin/Frequency of Three Times a Day Regimen dose: ------- NPH and regular or rapid before breakfast + regular or rapid before dinner + NPH at bedtime. Basal Bolus Regimen Dose: ------- Multiple dose. Type of Insulin/Frequency of Basal Bolus Regimen Dose: ------- Regular or rapid before breakfast, lunch and dinner + long acting once a day or regular or rapid before breakfast, lunch, and dinner + NPH twice a day.

Key Concept Summary 1. Serum glucose is maintained within a narrow range by the hormones insulin and glucagon. 1. The pancreas and liver mainly control glucose levels. 2. The pancreas secretes insulin, which decreases serum glucose levels, and glucagon, which increases glucose levels. 3. After a meal, insulin is secreted and glucose moves from the blood into body cells. 4. Between meals, hepatic glycogen is broken down to maintain a stable serum glucose level. 2. Type 1 diabetes is characterized by insufficient insulin synthesis by the pancreas, 1. whereas type 2 diabetes is characterized by insulin resistance in the target cells. 2. The two basic types of diabetes are type 1 and type 2 diabetes. 3. In type 1 diabetes there is a lack of insulin, so glucose levels increase and the body cannot use glucose for energy. 4. Type 2 diabetes results from insulin resistance, usually from insulin receptor site dysfunction, which also results in the inability of glucose to enter cells for energy. 5. Gestational diabetes has an onset, or first recognition during pregnancy, which places both the woman and baby at risk for complications. 3. The classic signs and symptoms of diabetes include polyuria, polydipsia, and polyphagia. 1. The signs and symptoms of diabetes may develop rapidly, as occurs with type 1, or more gradually, as is seen with type 2 diabetes. 2. The classic symptoms are polyuria, polydipsia, and polyphagia. T 3. he two primary diagnostic tests used to diagnose diabetes are the fasting plasma glucose test, and the OGTT. 4. Based on the results, a person may be diagnosed as having diabetes or prediabetes.

4. Acute complications of diabetes include hypoglycemia, diabetic ketoacidosis (DKA), and hyperosmolar hyperglycemic state. 1. Hypoglycemia can develop quickly in patients treated with insulin. 2. Hypoglycemia usually responds quickly to administration of a concentrated glucose source but glucagon can also be given to raise the glucose levels. 3. DKA is marked by hyperglycemia, osmotic diuresis with dehydration and electrolyte imbalances, and ketoacidosis. 4. Treatment includes fluid and electrolyte replacement, and normalization of glucose levels. 5. HHS involves extremely high glucose levels, hyperosmolarity, dehydration, and CNS dysfunction. 6. Treatment consists of fluid replacement, correction of electrolyte imbalances, and insulin to reduce the elevated glucose levels. 5. Serious, chronic complications of diabetes include neuropathy, nephropathy, retinopathy, and vascular disease. 1. Aggressive control of blood glucose can lower the risk for long-term consequences of diabetes. 2. These include neuropathy, nephropathy, retinopathy, and vascular disease. 3. Common chronic complications include paresthesias, renal failure, blindness, CVA, and amputations. 6. Insulin is the cornerstone of therapy for patients with type 1 and gestational diabetes. 1. Patients with type 1 diabetes require lifelong insulin replacement therapy. 2. Those with type 2 diabetes may achieve glucose control by diet and exercise alone, or may require an oral medication with or without insulin. 3. Types of insulin differ by onset and duration of action. 4. Many patients with type 1 diabetes take an intermediate-acting insulin for basal coverage, with additional injections of a rapid-acting insulin before meals. 5. The main adverse effect of insulin is hypoglycemia, which can occur quickly, and is most likely to occur at the time of peak action for that insulin. 7. Oral hypoglycemic drugs are used to treat type 2 diabetes. 1. There are several classes of oral drugs used to treat type 2 diabetes.

2. These oral agents may be used alone, or in combination with another oral agent or insulin. 3. Sulfonylureas stimulate the release of insulin from the pancreas, and increase the sensitivity of peripheral tissues to insulin. 4. Biguanides lower glucose levels by suppressing hepatic secretion of glucose and increasing cellular uptake of glucose. 5. Meglitinides increase insulin release from the pancreas. 6. Thiazolidinediones lower glucose levels by reducing insulin resistance, and decreasing gluconeogenesis in the liver. 7. Alpha-glucosidase inhibitors slow carbohydrate metabolism in the intestines, lessening the rise in postprandial glucose seen following a meal. 8. Incretin therapies offer a new approach to treating type 2 diabetes. 1. Newer therapies include drugs that affect incretins, hormones that increase insulin secretion by the pancreas. 2. Incretin mimetics such as exenatide have the same actions as endogenous incretins. 3. DDP-IV inhibitors such as sitagliptin decrease the breakdown of incretins. 4. They also increase satiety, resulting in lower calorie intake and weight loss.

Chapter 67 Key Terms a type of metabolic acidosis due to an excess of ketone diabetic bodies, most often occurring when diabetes mellitus is ketoacidosis uncontrolled (DKA) fasting plasma glucose(FPG) test gestational diabetes a primary blood test for diagnosing diabetes condition resulting from glucose intolerance with an onset, or first recognition, during pregnancy

the production of "new" glucose from noncarbohydrate gluconeogenesis molecules glycogenolysis the process of glycogen breaking down

a laboratory test used in diabetic management; also hemoglobin A1C called glycosylated hemoglobin hyperosmolar hyperglycemic state (HHS) incretin enhancer acute complication seen in persons with type 2 diabetes, that is characterized by extreme hyperglycemia, hyperosmolarity with dehydration, the absence of ketoacidosis, and CNS dysfunction. synthetic drug that augments natural glucose-lowering hormones by preventing the breakdown of incretins

synthetic drug that augments natural glucose-lowering incretin mimetic hormones that has all the actions of the natural drug insulin insulin resistance metabolic syndrome oral glucose tolerance test (OGTT) polydipsia polyphagia pancreatic hormone that acts to decrease blood glucose levels occurs in type 2 diabetes mellitus; insulin receptors in target tissues become insensitive to insulin a group of abnormalities that tend to occur together that places the patient at increased risk for developing diabetes and vascular complications disease, and CVA a primary blood test for diagnosing diabetes excessive thirst excessive appetite

polyuria prediabetes

excessive urine production condition in which serum glucose levels are elevated but are not high enough to be diagnosed as diabetes

metabolic disease characterized by hyperglycemia type 1 diabetes caused by a lack of secretion of insulin by the pancreas chronic metabolic disease caused by insufficient type 2 diabetes secretion of insulin by the pancreas, and a lack of sensitivity of insulin receptors

1.

During insulin therapy, nursing interventions to reduce the risk for lipodystrophy include: Select all that apply.

Correct Rotating injection sites. Answers: Avoiding using any one injection site more often than once a month. Correct. Rationale: Rotating injection sites and avoiding the use of any one site more often than once a month decrease the risk for lipodystrophy. Monitoring glucose levels (option 1) and assessing sensation (option 3) are both important, but would not alter the risk for lipodystrophy. The client should always use needles specifically designated for insulin use to administer their insulin (option 5).

2.

Before administering a morning lispro insulin injection, the nurse should: Select all that apply.

Correct Check that mornings fasting glucose level. Answers: Ensure that breakfast trays are present on the unit. Assess for symptoms of hypoglycemia. Rationale: It is always necessary to review the latest glucose level prior to administering any type of insulin (option 2).

Because lispro is a rapid-acting insulin, the nurse should ensure that the client will be able to eat a meal shortly after receiving a dose (option 3), and that no signs of hypoglycemia are present (option 5). It is not necessary to check a urine sample for glucose or ketones (option 1). Urine testing does not give exact information, and clients vary on the degree to which glucose and ketones will spill into the urine. It is not necessary to check a pulse or blood pressure prior to giving insulin, as insulin does not directly affect these (option 4).

3.

The nurse would attribute which of the following assessment findings as adverse effects to metformin therapy? Your Lactic acidosis Answer: A serious adverse effect of metformin is the risk for developing lactic acidosis. Renal insufficiency and failure, excess alcohol use, IV contrast agents increase the risk for lactic acidosis, and so are contraindications to its use.

The other adverse effects listed, hypoglycemia (option 1), GI distress (option 2), and weight loss (option 4) are adverse effects common among oral hypoglycemic drugs.

4.

A client was started on rosiglitazone for type 2 diabetes. He tells the nurse that he has been taking it for 5 days, but his glucose levels are unchanged. The nurses best response is: You need to give the drug more time. It can take several weeks before it becomes effective. It can take several weeks for this drug to provide therapeutic

effects, so the appropriate response would be to give this drug more time to reach effectiveness. It would not be appropriate for the nurse to change a clients dose (option 1), add a second drug (option 3), or tell the client he will be needing insulin (option 4).

5.

A young woman calls the clinic and reports that her mother had an insulin reaction, and was found unconscious. The young woman gave her a glucagon injection 20 minutes ago, and her mother woke up, but is still groggy and does not make sense. The nurse should tell her: Give her another injection and call the paramedics. Glucagon injections can be repeated if one dose is not effective. Hypoglycemia is a medical emergency, and since this woman has not fully recovered, medical attention is needed. It would not be appropriate to take no action (option 1). It would not be safe to give this client anything by mouth yet, as she is still groggy, and not making sense (option 2). Finally, it would not be normal or expected for a person to remain sleepy following recovery from hypoglycemia (option 3).

6.

The nurse explains to the client that the benefit of using the long-acting insulin glargine over other insulins is that it: Has no definite peak. This insulin has no definite peak, so there is a minimal risk for hypoglycemic reaction. It does have to be given by subcutaneous injection (option 1), cannot be given by IM injection (option 2), blood glucose monitoring is required for all clients taking any insulin (option 3).

7.

A postoperative client with type 1 diabetes is currently being treated with prednisone, a glucocorticoid, to reduce inflammation. The nurse

anticipates that this clients insulin therapy needs will be: Higher than before. Both the stress of surgery and glucocorticoids such as prednisone increase glucose levels. Postoperative clients and those taking glucocorticoids often require higher doses of insulin or oral hypoglycemic agents.

8.

The nurse would question an order for a sulfonylurea drug for which of the following clients?

One who is allergic to sulfa drugs There is a cross-sensitivity between sulfonylureas and sulfa drugs and thiazide diuretics, so they are contraindicated in clients allergic to either of these. Sulfonylureas can be combined with metformin and taken as a combination drug (option 1). Renal failure (option 2), and pregnancy (option 4) require cautious use of sulfonylureas, but are not contraindications.

9.

Which of the following does the nurse recognize as a recommended diagnostic test for long-term rosiglitazone therapy? Periodic liver function tests There has been concern about rosiglitazone causing liver damage because an earlier drug in this drug class was withdrawn from the market due to liver failure in some clients who were taking it. There is no direct evidence that this drug causes damage, but it is recommended that liver function tests be monitored. CBCs (option 1), electrocardiograms (option 2), and blood gases (option 4) would not provide information on liver function.

1 0.

Following a subcutaneous injection of lispro insulin, during what period should the nurse monitor the client for the greatest likelihood of hypoglycemic reaction?

Correct 11.5 hours Answer:

The time of greatest risk for hypoglycemic reaction comes at the time of peak action for any insulin. For lispro insulin, this peak comes at 1-1.5 hours following the injection.

1 .

How would you, the nurse, explain diabetes mellitus to Jasons parents using simple terminology?

When describing diabetes mellitus to this patients parents, it may be helpful to convey the information that will provide them with a mental picture. For example, it may be helpful to visualize insulin as a gatekeeper. When present, insulin swings open the gate, transporting glucose inside the cell. However, if there is no insulin, there is not entry and the body uses other sources for energy.

2 .

What will likely be included in the treatment plan for Jason?

The nurse anticipates the following treatment: admission to a critical care area; IV placement, and infusion of 0.9% saline to rehydrate the patient; regular insulin, beginning with an IV bolus, followed by a slow, continuous IV infusion until glucose levels are 250 mg/dL or lower; electrolyte and acidosis correction; and frequent laboratory testing of serum glucose and electrolyte levels.

3 .

What will be the immediate teaching needs of Jason and his parents?

The patient and his parents will need extensive teaching on diabetes and its management. Teaching can be organized and directed by a diabetic nurse educator if one is available. The parents will need to understand the disorder, the need for lifelong management, and the possibility of complications, both acute and chronic. They need to understand the relationships between diet, exercise, medications, and glucose levels. They need to understand that glucose levels and insulin needs can vary during times of stress (illness, injury), during growth spurts, and after periods of unusual exercise. They also need medication teaching for insulin. This will include the actions of the drug, and times of onset of action, peak action, and duration of action, how to administer subcutaneous injections and the sites used, and how to store the medication. They will need a registered dietitian to help develop an individualized dietary plan for Jason. They also need to know how to monitor glucose levels using a home glucose meter, and the timing of the

insulin injections. They must be taught the signs of hypoglycemia, and the appropriate actions to take should it occur. They should be taught to inform the health care provider of episodes of hypoglycemia so that adjustments in medication can be made. Jason and his parents will need nursing support during this major time of adjustment. Case Study #1 A 48-year-old male office manager is seen at a health clinic for his annual physical examination. He is moderately overweight, obtains little or no regular exercise, and eats fast food frequently due to job demands. He has no history of any serious medical conditions, and currently is having no unusual symptoms. He does not smoke or drink alcohol. His blood pressure is 132/86 mmHg; waist circumference 112 cm (44 in.); laboratory results show that his CBC, thyroid, and liver function tests are within normal limits; the lipid profile results are HDL 34 mg/dL, LDL 128 mg/dL, triglycerides 274 mg/dL; fasting glucose 116 mg/dL.

What is the most likely cause of this patients laboratory results? These results most likely represent metabolic syndrome. What diabetes classification best describes this patient? This patient would be classified as having prediabetes. What type of treatment is appropriate for this patient? What are his teaching needs? Having prediabetes places the patient at increased risk for developing type 2 diabetes. In addition, the metabolic syndrome places him at increased risk for developing type 2 diabetes, and vascular complications such as cardiovascular disease, peripheral vascular disease, and stroke. He needs to understand these risks, and that he can take actions to delay or prevent the progression to type 2 diabetes. These actions include obtaining regular exercise, maintaining a normal or near-normal weight and blood pressure, and eating a lower fat, lower calorie diet. Patient teaching should include each of these points, and how they can be accomplished by the patient. sulfonylureas -------------- oral hypoglycemic. stimulate the release of insulin from the pancreas. increase sensitivity of insulin receptors. adverse effects include hypoglycemia, weight gain, GI distress, hypersensitivity reactions, hepatotoxicity. can cause hypoglycemia biguinides -------------- oral hypoglycemic. decreases hepatic production of glucose. reduces insulin resistance. promotes weight loss. adverse effects include anorexia, nausea, diarrhea

alpha-glucosidase inhibitors -------------- oral hypoglycemic. block enzymes responsible for breaking down complex carbohydrates. adverse effects include abdominal pain, cramping, diarrhea, flatulence. can cause hypoglycemia thiazolidinediones -------------- oral hypoglycemic. also known as glitazones. reduce blood glucose by decreasing insulin resistance and inhibiting hepatic gluconeogenesis. adverse effects are fluid retention, weight gain, headache. may be hepatotoxic. contraindicated in patients with heart failure or pulmonary edema. can cause hypoglycemia megalitinides -------------- oral hypoglycemic. stimulate release of insulin. short duration of action. can cause hypoglycemia. can cause hypoglycemia insulin aspart (novalog) -------------- rapid action. 10-20 min onset, subQ before meal. can give with NPH, should be drawn first. insulin lispro (humalog) -------------- rapid action. 5-15 min onset. subQ before meal. can give with NPH, should be drawn first insulin glulisine (apidra) -------------- rapid action. 15-30 min onset. subQ before meal. can give with NPH, should be drawn first insulin regular (humulin R, novolin R) -------------- shor action, 30-60 min onset, subQ before meal or IV, can mix with NPH, sterile water, saline. do NOT mix with glargine isophane susp (NPH, Humulin N) -------------- intermediate action, 1-2 h onset, subQ/mix (cloudy). can mix with aspart, lispro, or regular. do NOT mix with glargine insulin detemir (levemir) -------------- long action, gradual onset, subQ one or 2 a day, do NOT mix insulin glargine (lantus) -------------- long action, 1.1 h onset, subQ one a day same time every day. do NOT mix. lispro (humalog) -------- rapid acting insulin lispro humalog -------- onset 10-15 minutes lispro humalog -------- peak 1 hour lispro humalog -------- duration 2-4 hours lispro humalog -------- used for rapid reduction of glucose level, to treat postprandial hyperglycemia, and/or to prevent nocturnal hypoglycemia

aspart novolog -------- rapid acting insulin aspart novolog -------- onset 5-15 min aspart novolog -------- peak 40-50 min aspart novolog -------- duration 2-4 hours aspart novolog -------- used for rapid reduction of glucose level, to treat postprandial hyperglycemia, and/or to prevent nocturnal hypoglycemia glulisine apidra -------- rapid acting insulin glulisine apidra -------- onset 5-15 mins glulisine apidra -------- duration 2 hours glulisine apidra -------- used for rapid reduction of glucose level, to treat postprandial hyperglycemia, and/or to prevent nocturnal hypoglycemia short acting insulins -------- regular (Humalog R, Novolin R, Iletin II Regular) regular (Humalog R, Novolin R, Iletin II Regular) -------- onset 1/2-1 hours regular (Humalog R, Novolin R, Iletin II Regular) -------- peak 2-3 h regular (Humalog R, Novolin R, Iletin II Regular) -------- duration 4-6 hours regular (Humalog R, Novolin R, Iletin II Regular) -------- usually administered 20-30 minutes before a meal regular (Humalog R, Novolin R, Iletin II Regular) -------- may be taken alone or in combination with longer-acting insulin Intermediate acting insulin -------- NPH NPH -------- onset 2-4 hours NPH -------- peak 4-12 hours NPH -------- duration 16-20 hour NPH -------- usually taken after food

Humulin N, Iletin II Lente, Iletin II NPH, Novolin L (Lente), Novolin N (NPH) -------- onset 3-4 hours Humulin N, Iletin II Lente, Iletin II NPH, Novolin L (Lente), Novolin N (NPH) -------- peak 412 hours Humulin N, Iletin II Lente, Iletin II NPH, Novolin L (Lente), Novolin N (NPH) -------- duration 16-20 hours Humulin N, Iletin II Lente, Iletin II NPH, Novolin L (Lente), Novolin N (NPH) -------- usually taken after food very long acting insulins -------- Glargine (Lantus) Detemir (Levemir) Glargine (Lantus) Detemir (Levemir) -------- onset 1 hour Glargine (Lantus) Detemir (Levemir) -------- continuous (no peak) Glargine (Lantus) Detemir (Levemir) -------- duration 24 hours Glargine (Lantus) Detemir (Levemir) -------- used for basal dose of insulin 3 causes of morning hyperglycemia -------- insulin waning dawn phenomenon somogyi effect insulin waning -------- progressive rise in blood glucose from bedtime to morning treatment for insulin waning -------- increase evening (predinner or bedtime) dose of intermediate acting or long acting insulin, or institute a dose of insulin before the evening meal if one is not already part of the treatment regimen dawn phenomenon -------- relatively normal blood glucose until about 3am, when the level begins to rise treatment for dawn phenomenon -------- change the time of injection of evening intermediate acting insulin from dinnertime to bedtime

somogyi effect -------- normal or elevated blood glucose at bedtime, a decrease at 2-3am to hypoglycemic levels and a subsequent increase caused by the production of counterregulatory hormones treatment of somogyi effect -------- decrease evening (predinner or bedtime) dose of intermediate acting insulin, or increase bedtime snack first generation sulfonylureas -------- acetohexamide (Dymelor) chlorpropamide (Diabinese) tolazamide (tolinase) Tolbutamide (orinase)

First generation sulfonylureas action/indications acetohexamide (Dymelor) chlorpropamide (Diabinese) tolazamide (tolinase) Tolbutamide (orinase) -------- action/indications *used in type 2 diabetes to control blood glucose levels *stimulates beta cells of the pancreas to secrete insulin *may improve binding between insulin and insulin receptors or increase the number of insulin receptors side effects of first generation sulfonylureas acetohexamide (Dymelor) chlorpropamide (Diabinese) tolazamide (tolinase) Tolbutamide (orinase) -------- SE *hypoglycemia *mild GI symptoms *weight gain *drug-drug interactions (NSAIDS, warfarin, sulfonamides) *sulfa allergy *skin reactions implications of first generation sulfonylureas acetohexamide (Dymelor) chlorpropamide (Diabinese) tolazamide (tolinase) Tolbutamide (orinase) -------- implications *monitor pt for hypoglycemia *monitor blood glucose and urine ketone levels to assess effectiveness of therapy implications of first generation sulfonylureas

acetohexamide (Dymelor) chlorpropamide (Diabinese) tolazamide (tolinase) Tolbutamide (orinase) -------- implications *patients at high risk for hypoglycemia: advanced age, renal insufficiency *When taken wih beta-adrenergic blocking agents, may mask usual warning signs and symptoms of hypoglycemia implications of first generation sulfonylureas acetohexamide (Dymelor) chlorpropamide (Diabinese) tolazamide (tolinase) Tolbutamide (orinase) -------- implications *instruct patients to avoid the use of alcohol *check for interactions with other meds

second generation sulfonylureas -------- *Glipizide (glocotrol, glocotrol XL) *Glyburide (Micronase, Glynase, Dia-Beta) *Glimepiride (Amaryl) second generation sulfonylureas action/indications *Glipizide (glocotrol, glocotrol XL) *Glyburide (Micronase, Glynase, Dia-Beta) *Glimepiride (Amaryl) -------- action/indications *stimulates beta cells to secrete insulin *may improve binding between insulin and insulin receptors or increase the number of insulin receptors) *Used in type 2 *more potent effects than 1st gen *Glipizide (glocotrol, glocotrol XL) *Glyburide (Micronase, Glynase, Dia-Beta) *Glimepiride (Amaryl) action/indications -------- action/indications May be used in combination with metformin or insulin to improve glucose control *Glipizide (glocotrol, glocotrol XL) *Glyburide (Micronase, Glynase, Dia-Beta) *Glimepiride (Amaryl) 2nd gen sulfonylureas side effects -------- side effects *Hypoglycemia *mild GI symptoms

*Weight gain *Drug-drug interactions (NSAIDS, warfarin, sulfonamides) *sulfa allergies *Glipizide (glocotrol, glocotrol XL) *Glyburide (Micronase, Glynase, Dia-Beta) *Glimepiride (Amaryl) 2nd gen sulfonylureas implications -------- implications *monitor pt for hypoglycemia *monitor blood glucose and urine ketone levels to assess effectiveness of therapy *patients at high risk for hypoglycemia: advanced age, renal insufficiency implications *Glipizide (glocotrol, glocotrol XL) *Glyburide (Micronase, Glynase, Dia-Beta) *Glimepiride (Amaryl) 2nd gen sulfonylureas -------- implications *when taken with beta blockers: may mask usual warning signs and symptoms of hypoglycemia *instruct patients to avoid the use of alcohol biguanides -------- Metformin (Glucophage, glucophage XL, Fortamet) Metformin with glyburide (glucovance) Metformin (Glucophage, glucophage XL, Fortamet) Metformin with glyburide (glucovance) action/indications -------- action/indications *inhibits production of glucose by the liver *increases body tissue's sensitivity to insulin *decreases hepatic synthesis of cholesterol *used in type 2 to control blood glucose level Metformin (Glucophage, glucophage XL, Fortamet) Metformin with glyburide (glucovance) side effects -------- side effects lactic acidosis Metformin (Glucophage, glucophage XL, Fortamet) Metformin with glyburide (glucovance) side effects -------- side effects hypoglycemia if used in combination with insulin or other antidiabetic agents drug-drug interactions

Metformin (Glucophage, glucophage XL, Fortamet) Metformin with glyburide (glucovance) side effects -------- side effects contraindicated in patients with impaired renal or liver function, respiratory insufficiency, severe infection, or alcohol abuse Metformin (Glucophage, glucophage XL, Fortamet) Metformin with glyburide (glucovance) implications -------- implications monitor for lactic acidosis and hypoglycemia Metformin (Glucophage, glucophage XL, Fortamet) Metformin with glyburide (glucovance) implications -------- implications *monitor renal function *patients taking this drug are at increased risk of acute renal failure and lactic acidosis with use of iodinated contrast material for diagnostic studies Metformin (Glucophage, glucophage XL, Fortamet) Metformin with glyburide (glucovance) implications -------- implications should be stopped 48 hours before and after use of contrast agent or until renal function is evaluated and is normal alpha-glucosidase inhibitors -------- acarbose (precose) miglitol (glyset) acarbose (precose miglitol (glyset) action/indication -------- action/indications delays absorption of complex carbs in the intestine and slows entry of glucose into systemic circulation acarbose (precose) miglitol (glyset) action/indication -------- action/indications does not increase insulin secretion used in type 2 can be used alone or in combination with sulfonylureas, metformin, or insulin to improve glucose control acarbose (precose) miglitol (glyset)

side effects -------- GI side effects: abdominal discomfort or distention, flatulence, diarrhea acarbose (precose) miglitol (glyset) side effects -------- side effects hypoglycemia: risk increased if used with insulin or other antidiabetic agents acarbose (precose) miglitol (glyset) implications -------- implications must be taken with the first bite of food to be effective acarbose (precose) miglitol (glyset) implications -------- implications monitor for GI side effects: diarrhea, abdominal distention) acarbose (precose) miglitol (glyset) implications -------- implications monitor for blood glucose levels to assess effectiveness of therapy acarbose (precose) miglitol (glyset) implications -------- implications monitor liver function studies every 3 months for 1 year, then periodically acarbose (precose) miglitol (glyset) implications -------- implications contraindicated in patients with GI or renal dysfunction or cirrhosis acarbose (precose) miglitol (glyset) implications -------- implications Alert: hypoglycemia must be treated with glucose, not sucrose non-sulfonylurea insulin secretagogues -------- repaglinide (prandin) categorized as a meglitinide nateglinide (starlix) categorized as a d-phenylalanine derivative repaglinide (prandin) categorized as a meglitinide nateglinide (starlix) categorized as a d-phenylalanine derivative

action/indications -------- action/indications *stimulates pancreas to secrete insulin *used in type 2 to control blood glucose levels repaglinide (prandin) categorized as a meglitinide nateglinide (starlix) categorized as a d-phenylalanine derivative action/indications -------- action/indications *can be used alone or in combination with metformin, or thiazolidinediones to improve glucose control repaglinide (prandin) categorized as a meglitinide nateglinide (starlix) categorized as a d-phenylalanine derivative side effects -------- side effects hypoglycemia/weight gain less likely than sulfonylureas repaglinide (prandin) categorized as a meglitinide nateglinide (starlix) categorized as a d-phenylalanine derivative side effects -------- side effects drug-drug interactions with ketoconazole, fluconazole, erythromycin, rifampin, isoniazid repaglinide (prandin) categorized as a meglitinide nateglinide (starlix) categorized as a d-phenylalanine derivative implications -------- implications *monitor blood glucose levels to assess effectiveness of therapy *has rapid action and short half-life repaglinide (prandin) categorized as a meglitinide nateglinide (starlix) categorized as a d-phenylalanine derivative implications -------- implications *should be taken only if able to eat a meal immediately *teach patients symptoms of hypoglycemia repaglinide (prandin) categorized as a meglitinide nateglinide (starlix) categorized as a d-phenylalanine derivative implications -------- implications *monitor pts with impaired liver function and renal impairment *has no effect on plasma lipids repaglinide (prandin) categorized as a meglitinide nateglinide (starlix) categorized as a d-phenylalanine derivative implications -------- implications *is taken before each meal *check for interactions with other meds

thiazolidinediones (glitazones) -------- pioglitazone (actos) rosiglitazone (avandia) pioglitazone (actos) rosiglitazone (avandia) side effects -------- side effects *sensitize body tissue to insulin *stimulate insulin receptor sites to lower blood glucose and improve action of insulin pioglitazone (actos) rosiglitazone (avandia) side effects -------- side effects *may be used alone or in combination with sulfonylurea, metformin, or insulin pioglitazone (actos) rosiglitazone (avandia) side effects -------- side effects *hypoglycemia (risk increased with use of insulin or other antidiabetic drugs) *anemia pioglitazone (actos) rosiglitazone (avandia) side effects -------- side effects *weight gain *edema *decreases effectiveness of oral contraceptives pioglitazone (actos) rosiglitazone (avandia) side effects -------- side effects *possible liver dysfunction *drug-drug interaction pioglitazone (actos) rosiglitazone (avandia) side effects -------- side effects *hyperlipidemia: has variable effect on lipids *impaired platelet function pioglitazone (actos) side effects -------- side effects may be preferred choice in patients with lipid abnormalities pioglitazone (actos)

rosiglitazone (avandia) implications -------- implications *monitor blood glucose levels to assess effectiveness of therapy *monitor liver function tests *arrange dietary teaching to establish weight control program *instruct pts taking oral contraceptives about increased risk of pregnancy dipeptidyl peptidase 4 (DPP4) inhibitor -------- sitagliptin (januvia) vildagliptin (galvus) sitagliptin (januvia) vildagliptin (galvus) action/indications -------- implications increases and prolongs the action of incretin, a hormone that increases insulin release and decreases glucagon levles, with the result of improved glucose control sitagliptin (januvia) vildagliptin (galvus) side effects -------- SE *upper respiratory infection *stuffy or sunny nose and sore throat sitagliptin (januvia) vildagliptin (galvus) side effects -------- SE *headache *stomach discomfort and diarrhea *hypoglycemia, if used with sulfonylurea sitagliptin (januvia) vildagliptin (galvus) implications -------- implications *usually administered once a day *used alone or with other oral antidiabetic agents *instruct patient about signs and symptoms of hypoglycemia and other adverse effects to report *monitor renal function

Type ____ Diabetes has a high incidence of DKA. While Type _____ Diabetes has a higher incidence of MI and Stroke. --------- Type I- DKA Type II- stroke and MI

Type _____ diabetes runs the risk to develop HHCN- hyperosmolar hyperglycemic nonketotic coma. --------- Type II- HHCN- still a little bit of insulin secreted= no ketones Type I instead develops DKA cuz there is NO INSULIN at all. Which two names are associated with the SYNTHETIC insulins- a.k.a Insulin Analog? --------- Humalog, and Novolog Fast onset & Shorter duration than regular insulin. *Must use a long acting insuling with it!!!!! Feed your pt before giving these insulins or u will see-Hypoglycemia. What types of events may increase a diabetics glucose level? --------- illness, infection, surgery, & stress! So I'm assuming that nursing school is contraindicated for Diabetics (we r so proud of Elizabeth and Cami!- defying the odds! lol) This oral hyperglycemic drug family is known as the 'oldest and largest' group of oral agents. Their method of action is to INCREASE secretion of insulin by stimulating the beta cells and they may also increase tissue responsiveness to insulin. Which oral agent family is this?? --------- Sulfonylureas!! - all four drugs listed in the syllabus end in the suffix -ide. (dont get confused with meglitinides though) This family of oral hyperglycemic agents have 2 drugs in it! They aren't used that often though, because they make the pt fart like crazy! (not a joke) & they're expensive! (who would pay a lot of money to have gas pains?) Their method of action is to delay the digestion of carbs when the drug and food are in the GI tract @ the same time. Which oral agent family is this? --------- Alpha-glucosidase Inhibitors- $$$$ and used infrequently because of flatulence side effect. No Beano=No Bueno! - Memory trick! This oral hyperglycemic agent family has just one drug in it- its an only child and its perfect- (according to Nancy) This drug does NOT increase appetite and therefore no weight gain! It also makes muscle and fat cells use up more glucose! It does cause cramps and diarrhea- so start low & go slow- and if youre over 80- sorry partner- its NO go for u. Which oral agent family is this? --------- Biguannide-- drug is metformin (Glucophage) _ this drug does NOT cause hypoglycemia and also decreases hepatic production of glucose. This oral hyperglycemic agent family has 2 drugs in it! They are known for DECREASING INSULIN RESISTANCE!! woo hoo. However they are very expensive & sulfonylureas work just as well. The worst S/E is possibly making heart failure worse by increasing pedal edema! Which oral agent family is this? --------- Glitazones! the drugs are ACTOS & AVANDIA * taking one of these drugs with metformin (Glucophage) gives you tighter glucose control. This oral hyperglycemic agent family has 2 drugs in it also. This class is the same in action as the Sulfonylureas- they INCREASE PANCREATIC SECRETION OF INSULIN. So both of those classes are known as Secretagogues. I can however, cause hypoglycemia & I cost a lot of $$. If you skip a meal, skip a pill. If you add a meal, add a pill!

Which oral agent family is this? --------- Meglitinide- drugs are PRANDIN & STARLIX This oral hyperglycemic drug does not have any other drugs in its family- its a single bachelor-(new on the market). This drugs claim to fame is that it is a protein derived from the saliva of a Gila monster. ( & this ain't no joke either). This drug is almost like a dream drug for Type II diabetics: it stimulates the beta cells to release the amount of insulin needed in proportion to the amt of glucose present! It suppressed glucagon production in the liver and suppressed gastric emptying to promote a feeling of satisfaction in the brain!= weight loss! It is given SQ twice daily and costs about $200 a month! Which oral drug is this? --------- Exenatide (Byetta) This drug can also be used in combination with metformin & sulfonylureas. This oral agent is given to weight neutral Diabetics who do not need to lose an excessive amount of weight. It is effective in lowering the Hb A1C levels, similar to Byetta. This drug also does NOT cause any problems with hypoglycemia. Which oral agent is this? --------- Sitagliptin (Januvia) Use as adjunct to Metformin or Actos Combination therapy is used when diet, exercise and single drug therapy are not adequately controlling glucose levels. Finish the following combinations: Insulin+ (2 oral agent classes)? Sulfonylureas + (3 oral agent classes)? Metformin + (2 oral drug classes and sitagliptin) --------- INSULIN + sulfonylurea OR pioglitazone SULFONYLUREA + acarbose/miglitol (alpha-gluc. inhib) SULFONYLUREA + metformin OR glitazone METFORMIN + repaglinide OR a glitazone OR sitagliptin Exercise is extremely important for Diabetic patients. Exercise has two main benefits for the Diabetic. The nurse should teach their pt that exercise ________________ & ________________; helping decrease long term complications from diabetes. --------Exercise : 1- helps the body tissues use insulin more proficiently. & 2- helps glucose move out of the blood stream and into the muslces= lower glucose levels! BOTH of these decrease the long-term complications of Diabetes. What are the three "P's" associated with Hyperglycemia? --------- Polyuria--polydipsia-and polyphagia What is the treatment for a CONSCIOUS patient experiencing Hypoglycemia? and what about UNCONSCIOUS? --------- conscious: Give them 15 grams of carbohydratewait 15-25 mnutes and recheck BS. Repeat if blood sugar does not return to normal. Follow up with a protein snack. Unconscious: have family member or roomate inject Glucagon if @ homeIn the hospital a 25%-50% IV glucose solution is used. 15 grams of carbohydrate is equal to: _____ oz. of fruit juice/soda, ____ glucose tabs, ____ cup of skim milk. --------- 15 grams of carbs = 4 oz of juice/soda, 3 glucose tabs, and 1 cup of skim milk.

What site is insulin absorbed the fastest from? --------- the abdomenfollowed by the deltoid, thigh and hip. Where is the best place for a Diabetic who plans on exercising to inject their insulin? --------- The best place to inject insuling is a site away from the muscles to be used while exercising. For a diabetic, their goal A1C level should be ________. --------- Less than 7- (they had a 3 month average of 150 or less on their blood sugar). Which antihypertensive medication has protective effects on the kidneys for pts with type I and II diabetes? --------- ACE inhibitors- they protect the kidneys in MORE than one way. Meaning: aside from lowering B/P, which also helps the kidney, it does something more to protect them. Hypoglycemia is defined as _____ mg/dL blood sugar reading. Name the S/S of Hypoglycemia. --------- <70 mg/dL blood sugar= hypoglycemia S/S: shaking, tachycardia, sweating, anxious, dizziness, hunger, blurred vision, weakness/fatigue, headache, irritable. Hyperglycemia is defined as ____ mg/dL blood sugar reading. Name the S/S of Hyperglycemia. --------- >200 mg/dL= hyperglycemia S/S: extreme thirst, polyuria, dry skin, polyphagia, blurred vision, drowsiness, nausea. * teach pt to call Dr. is multiple readings are over 200-250 mg/dL. Name three compensatory mechanisms utilized by the body when DKA is happening. --------- 1- The body exchanges the excessive amount of H+ ions for intracellular K+ ions, trying to correct the acidemia, but unfortunately, hyperkalemia develops as a consequence. 2-The lungs try to compensate for the acidemia by increasing the depth and rate of respirations to blow off more CO2. aka Kussmaul Respirations. 3. The kidneys excrete the ketone bodies through the urine which creates acetone in the urine. What are the clinical S/S of DKA and when is a Diabetic at risk for developing this condition? --------- S/S: early on they are Blurred vision, anorexia, N/V, thirst and polyuria. Late ones include drowsiness, progressing to stupor and coma, Kussmaul's resp., dehydration and other signs of fluid and electrolyte imbalances, hypotension, tachycardia, and other signs of shock. At risk: when insulin is omitted or illness such as infections, trauma, myocardial infections or stroke. What are the clinical manifestations of HHNC? (Hyperosmolar Hyperglycemic Nonketotic Coma) --------- This is another type of Diabetic coma that is more rare and carries a high mortality rate. Hyperosmolar refers to the excessive amt of glucose, electrolytes and other solutes in the blood in relation to the amt of water. S/S are hyperglycemia which leads to osmotic diruesis (polyuria) and resultant polydipsia, dehydration and electrolyte losses. Neuro S/S are coma, stupor and drowsiness. Additional S/S include hypovolemic shock, thrombosis, renal problems and stroke.

What type of patient is @ risk for developing HHNC? --------- The type II diabetic that had a mild or even unknown case of diabetes; usually after an illness. Occurs in Hyperglycemic conditions other than diabetes like severe burns, corticosteroid therapy. HHNC does NOT cause ketosis-like DKA does- because there is still insulin present so ketones are not formed. _________ or _________ is the insulin of choice for patients experiencing acute or emergency situations, DKA, HHNC, severe infections, other illness, major surgery and pregnancy. These insulins are also the only type available to be given through IV. --------Humulin R & Novolin R- "Short-acting insulins" _______, _____ & _______ are commonly used for long-term administration. They are a suspension with a cloudy appearance, that should only be rolled, NEVER shaken to mix up! Hypoglycemia is more likely to occur during mid to late afternoon. Usually given SQ 1-2 times daily. --------- NPH, Humulin N & Novolin N- "Intermediate-acting insulins" Insulin Lispro, a.k.a _______ & insulin aspart, a.k.a _______ are synthetic insulins with a faster onset and a shorter duration than the short-acting insulins, (regular). They are usually intended for use with another longer-acting insulin. These insulins are also great to be used in the 'external insulin pumps' (Like Cami told us about.) --------- Insulin Lisproa.k.a: HUMALOG & insulin aspart-a.k.a NOVOLOG These are both known as Insulin Analogs _______ is a long-acting insulin given once daily at bedtime. It is given to maintain a stable basal amount of insulin. It can never be mixed in the same syringe with another insulin! This dilutes it and it is a big NO NO! IT has a duration of 24 hours-hence the QD dosing. --------- Lantus- a.k.a insulin glargine _______ is also a long-acting insuling with once a day, and sometimes twice a day dosing. It is also given to provide a basal amount of insulin. Like Lantus, it too must never be mixed in the same syringe with another insulin. It has a shorter duration with more variability at 6-23 hours. This is why some pts can handle QD dosing while other require BID dosing. --------- Determir- a.k.a insulin levemir Name the hormones that normally RAISE blood glucose levels when hypoglycemia occurs. --------- Glucagon, epinephrine, growth hormone, and cortisol. What is a side effect associated with oral diabetic drug Metformin (glucophage) in older adults? --------- The development of lactic acidosis- potentially life threatening. Recommended smaller doses and monitoring of renal function. When giving Regular insulin to a patient, it is very important that the insulin be given ______ minutes before meals so that the insulin is available when blood sugar increases after meals. --------- 30-45 minutes before meals= reg insulin NOTE= when using an insulin analog like lispro or aspart insulin should be given 15 minutes before a meal. The patient that comes into the hospital with DKA and hyperkalemia will be started 1st on an IV solution, mainly ________. After insulin is on board and the patient's blood glucose levels come down, the pt will become ____________ and therefore should have K+

added to their IV solution and supplementation as appropriate. --------- IV solution- 0.9% Normal saline Hypokalemic.

Pharm Test 1: Endocrine Drugs ======= Pharm Test 1: Endocrine Drugs Diabetes Mellitus ======= - derived from the Greek work for fountain and the Latin word for honey - Disorder of carbohydrate metabolism that can proceed to fat and protein problems What is diabetes a disorder of ======= imbalance between insulin availability and insulin need Impact of diabetes as a disease ======= - affects approximately 24 million Americans - 7 million of those with diabetes - 79 million with prediabetes - in 20-74 years olds, leading cause of new blindness - leading 20-74 year olds, leading cause of new blindness - leading cause of kidney failure (44% of new cases in 2008) - 60-70% have mild to severe forms of nervous system damage - 1:400 children/adolescents have diabetes Classification of glucose disorders ======= - Clinical classes - Prediabetes classes Clinical classes of glucose disorders ======= * Type 1 * Type 2 * Gestational Prediabetes classes ======= * impaired glucose tolerance * Impaired fasting plasma glucose Type I diabetes ======= - absolute insulin deficiency - Destruction of the pancreatic beta cells - 5-10% of individuals with diabetes in US - Type 1A- immune mediated - Type 1B - idiopathic Characteristics ======= - absolute lack of insulin - elevated blood glucose - breakdown of fats and proteins for energy Signs and symptoms of Type 1 ======= - sudden onset - polyuria - poly dypsia - polyphagia - unexplained weight loss - ketonuria

- frequent infections - vision disturbances Type II Diabetes ======= - gradual onset, slow progression - 90-95% of diabetics in US - Older adults affected - recent increase in adolescents and children - Genetic and environmental factors Risk factors of Type II diabetes ======= - obesity - excess - abdominal fat Causes of Type II ======= - aging - Sedentary - Lifestyle - Genetic - Excess abdominal fat Patho of diabetes 2 ======= - peripheral insulin resistance - deranged insulin secretion of beta cells - increased production of glucose by liver Signs and symptoms of Type II diabetes ======= - may be asympomatic - More subtle: polydipsia, polyphagia (not common), polyuria, dry mouth - Obesity, weight gain - blurred vision - Skin infections - Fatigue - paresthesias Gestastional Diabetes ======= - disorder of glucose tolerance of variable severity, with onset during pregnancy - 2-14% of pregnancies affected Risk factors of Gestational Diabetes ======= - obesity - familial history - high birth weight offspring > 8.8 lbs, age Patho of gestational diabetes ======= - hormonal changes - increased cortisol (makes glucose available in the blood) - hyperglycemia in mother stimulates fetal insulin secretion Signs and symptoms of gestational diabetes ======= - mild polydipsia - polyuria - polyphagia Consequences of gestational diabetes ======= - metabolic abnormalities - stillbirth Blood/ Urine tests ======= - fasting blood glucose test - Casual blood glucose test

Oral glucose tolerance test Capillary blood glucose monitoring Glycated hemoglobin testing (A1C) Urine ketones

fasting blood glucose test ======= < 100 is normal 100-126 is impaired fasting glucose >126 on two occasions is diagnostic for diabetes Casual blood glucose test ======= - coupled with capillary blood glucose monitoring - done with glucometer at home or hospital > 200 in presence of classic symptoms gives a referral to physician for further diagnosis Oral glucose tolerance test ======= - screening test to measure bodies ability to store glucose by removing it from the blood - given concentrated glucose solution and measure response over 1 hour and 2 hours - if level goes down, then at least some insulin has been released Capillary blood glucose monitoring ======= - done with glucometer at home or hospital Glycated hemoglobin testing (A1C) ======= - when hgb is in the circulation primarily, it will not contain glucose - after being exposed to high blood sugar levels over time it will become glycated hgb cell - these can be tested to see the level of exposure to high blood glucose < 6% is desirable (7% or higher needs to be assessed) Urine ketones ======= - not as common because of glucometer - particularly used for type 1 and gestational Diagnosis of diabetes ======= - symptoms and a casual plasma glucoes > 200 mg/dl - Fasting plasma glucose > 126 mg/dl - 2 hour postprandial plasma glucose > 200mg/dl during an oral glucose tolerance test Diagnosis of prediabetes ======= - impaired fasting glucose: 100-126 gm/dl - Impaired glucose tolerance: 2 hour result is 140-199 mg/dl Goal of diabetic treatment ======= - maintain blood glucose levels within acceptable range to prevent acute and long-term complications Type 1 Core treatment ======= - diet - insulin replacement Additional treatments: - exercise - CBG monitoring - ACE inhibitor or ARB to prevent nephropathy - simvistatin MICROvascular problems with diabetics ======= - leads retinopathy or retinal hemorrhage

- lead to nephropathy (kidney) - lead to sensory or motor neuropathy MACROvascular problems with diabetics ======= - heart disease - elevated blood pressures - stroke Type II core treatment ======= - diet - exercise Additional treatments - oral hypoglycemics - insulin Treatment for gestational diabetes ======= - diet - exercise - glucose/ ketone monitoring Additional treatments - insulin 5 M's of diabetes treatment/mgmt ======= - Meal planning - Movement - Monitoring - Medication (always for Type 1) - Management (prevent ongoing complications) Dietary Recommendations ======= - protein 15-20% of calories - polyunsaturated fat - 10% of caolories - Saturated fates < 10% of energy intake - Carbohydrates and monounsaturated fats: 60-70% of calories How much cholesterol is recommended ======= 300mg daily How many meals should diabetics eat ======= -total caloric intake spread evenly throughout the day - meals should be 4-5 hours apart Exercise benefits ======= - lowers blood glucose - decrease insulin requirements - decreases LDL, inreases HDL - Improves CV function - Reduces insulin resistance What is the problem with exercise and diabetes ======= - strenuous exercise can cause hypoglycemia - dietary intake or insulin adjustments may be needed Pharmacologic mgmt of diabetes ======= - tight glycemic control is used to reduce long- term complications - Insulins (type 1, type 2, gestational) - Oral antidiabetic agents (type II only)

Pharmacologic mgmt of type I ======= - insulin must be coordinated with caloric intake Pharmacologic mgmt of type 2 ======= stepwise approach Step 1 - lifestyle changes Step 2- one oral antidiabetic Step 3 - 2 oral antidiabetic Step 4a - 3 orals Step 4b - oral plus insulin Step 5 - insulin alone How does treatment of type 2 differ between a lean and obese patient ======= -Obese- metformin because normally insulin resistance and this can correct it - Lean patient - sulfonourea will increase insulin production Insulin ======= - Anabolic hormone - promotes conservation of energy - builds up energy stores Effects of insulin ======= - stimulates the uptake of glucose, amino acids, nucleotides and K+ into cells - Promotes synthesis of complex organic molecules - glucagon - proteins - triglycerides When is insulin used? ======= - used to treat all forms of diabetes - but not always first line treatment describe glycogenolysis ======= - when glycogen is broken down to make glucose Describe gluconeogenesis ======= - proteins are degraded to amino acids to make glucose - (done when glycogen storage is depleted) * fats are also converted to glycerol and fatty acids through this process too types of insulin ======= 7 types! 1. Lispro (humalog) 2. Glulisine (Apidra) 3. Aspart (Novolog0 4. Regular (Humulin R) (SQ use) 5. NPH (Humulin N, Novolin N) 6. Detemir (Levemir) 7. Glargine (Lantus) What is the onset of insulin ======= 1. Humalog = rapid onset, short duration 2. Glulisine = rapid onset, short duration 3. Novolog = rapid onset, short duration 4. Humulin R = moderate rapid onset, short duration 5. NPH (humulin N, Novolin N)- intermediate duration 6. Levemir - intermediate duration

7. Lantus - prolong duration, no definite peak What does NPH stand for? ======= Neutral Protamin Hagadorn - basically regular insulin mixed with protamine and prolongs the duration and delays the onset time Sliding scale administration of insulin ======= - MD orders - Combines glucose monitoring with insulin administration by protocol Insulin Administration ======= High Alert Drug for Med Errors! Routs of insulin administration ======= - all forms are sub-q - regular - sq, iv, im Lispro, Aspart, Glu...., Regular = IV Altenate forms of insuliin delivery ======= - pens, - pumps Insulin Injections ======= should rotate injection sites Complications of insulin ======= - allergic reactions - lipodystrophy - insulin resistance - hypoglycemia lipodystrophy ======= - altered deposits of subcutaneous fats - two types -- lipid atrophy -- lipohypertrophy = increase in fat synthesis or increase in tissue around an injection site Lipid atrophy ======= - destruction of surrounding adipose tissue - looks like little hollows where the fat was worn away around the area where insulin is administered - rare How do you prevent lipodystrophy ======= rotate injection sites! Which insulin is a suspension ======= NPH - you will know it is NPH because it is cloudy Insulin drug interactions ======= - HYPOglycemic agents - HYPERglycemic agents HYPOglycemic agents ======= 1. Sulfonylureas 2. Glinides 3. Beta-adrenergic blocking agents 4. Alcohol HYPERglycemic agents ======= 1. Thiazide diuretics

2. Glucocorticoids (carb/fat/protein metabolism and elevate blood sugar) 3. Sympathomimetics (mimic sympathetic activity and release cortisol that increases blood sugar) Interaction between beta-adrenergic blocking agent and insulin ======= - symptoms of hypoglycemic will be masked when the beta -adrenergic blocking agent is given - the blocking agent will block the symptoms that are typical for hypoglycemia - ======= ... How to maintain Tight Glucose control ======= - insulin dosage matched with insulin need - self-monitoring of glucose needed 3-5x/day Benefit of tight glucose control ======= - it can reduce mortality and morbidity associated with diabetes - especially in type 1 patients because reduces the MICRO/MACRO vascular problems Glycemic control ======= - premeal plasma glucose: 90-130mg/dL - peak postmeal plasma glucose: < 180mg/dL - Hemoglobin A1C < 7% Risk of glucose control ======= - more expensive to patient - difficult to manage - less convenient for the patient - does increase the risk of HYPOglycemia Conventional vs. Intensive conventional therapies ======= - Conventional uses NPH approx. twice a day - Intensive Conventional uses regular at each meal and Glargine (long acting) for night time - Intensive Conventional is a tighter control Where is an insulin pump connected ======= - to the subcutaneous tissue What kind of insulin can you use? ======= - regular or NPH - programmed to give insulin all day to maintain a basal rate - can give a bolus also if needed Hypoglycemia blood glucose levels ======= < 50mg/dl Potential causes of blood glucose levels ======= - too much insulin - medication changes - reduction in food - intake - vomiting - diarrhea - excessive alcohol - unusual, intense exercise - childbirth

Signs and Symptoms of Hypoglycemia ======= - pallor - tremor - tachycardia - palpitations - sweating - headache - confusion/agitation - drowsiness - fatigue - convulsion - coma - death Treatment of hypoglycemia ======= - fast acting oral sugar - IV glucose - Parenteral glucagon Why not table sugar? ======= because it is normally in the form of sucrose and has to be broken down to glucose first Your client is hypertensive, diabetic and is prescribed insulin and beta-blockers. What symptoms of hypoglycemia would be masked by taking the beta blocker? ======= sweating and tachycardia Other than diabetes,what is another use of insulin? ======= - treatment for HYPERkalemia - insulin causes potassium to shift from blood to cell Which type of insulin has no peak action? ======= - glargine Which types of insulin can be given IV? ======= - Regular - very rapid acting What are the injection sites for a SQ injection? ======= - abdomen (preferred b/c consistent absorption) - back of the arm - thigh - upper back Which insulin is a suspension and needs to be agitated before administration? ======= NPH How should unopened vials of insulin be stored? Once opened, how stored? ======= refrigerated but not frozen for unopened - room temp for 1 month or expiration after opened Which insulins can be mixed with NPH? ======= - regular - short acting When mixed which goes into the syringe first? ======= short acting

Your insulin vial says U-100 on it. What does this mean? ======= - 100 units per mL (U-500 = 500 untis per mL) these are the 2 concentrations Oral hypoglycemics ======= - sulfonylureas - glinides (meglitinides) - Biguanides - Alpha- glucosidase inhibitors - Thiazolidinediones (glitazones) - Gliptins Biguanide ======= Metformin (Glucophage) glucaphage XL is extended release - use for type II diabetics, PCOD * good for people with insulin resistance Action of Biguanide ======= - decreases production of glucose in liver - reduces glucose absorption in gut - sensitizes insulin receptors at target tissues --> increases glucose uptake Pharmacokinetics of Biguanide ======= - absorbed in small intestine, excreted unchanged in kidney Adverse effects of Biguanide ======= - GI disturbances - decreased absorption of B12 and folic acid - Lactic acidosis - be careful with patients with renal insufficient, heart failure or prone to lactic acidosis how does lactic acidosis occur with metformin ======= metformin inhibits the metabolism of lactic acid by the cells and causes a build up of L.A. Sulfonylureas ======= - 2 generations Uses = type 2 diabetes Action of Sulfonylureas ======= - stimulates releases of insulin from pancreas, - increase cells sensitivity to insulin Pharmacokinetics of sulfonylureas ======= - metabolized by liver - excreted in urine Adverse effects of sulfonylureas ======= - hypoglycemia - cardiac toxicity Drug interactions of sulfonylureas ======= - alcohol - beta blockers - NSAIDs - Sulfonamide antibiotics - Cimetidine Difference in 2 generations of sulfonylureas ======= 1st - prototype is orinase 2nd - prototype is glypizide (Glucotrol)

* 2nd gen is more potent and produces effects at much lower dosages * less complications and less drug interactions with 2nd gen Why is hypoglycemia not a problem with metformin, when compared to a sulfonylurea? ======= - because metformin doesn't cause increase insulin production -.... metformin is available in extended release formulation. Can this be crushed? ======= no If I crush metformin what pharmacokinetic process is influenced? ======= absorption Glinides (Meglitinides) ======= - Repaglinide (prandin) - Use = type 2 diabetes Action of Meglitinides ======= - stimulates release of insulin from pancreas Pharmacokinetics of Meglitinides ======= - rapid absorption and elimination - hepatic metabolism, bile excretion - half life is 1 hours Adverse effects of Meglitinides ======= hypoglycemia (because it influences insulin production) Patient education of Meglitinides ======= - eat within 30 minutes of taking drug to prevent hypoglycemia because half life is so short Drug Drug interaction of Meglitinides ======= - Lopid- lowers TG level and can inhibit the action of Prandin Thiazolidinediones (glitazones) ======= - Rosiglitazone (Avandia) - Uses = type 2 diabetes Action of Thiazolidinediones ======= - reduce glucose by decreasing insulin resistance Pharmacokinetics of Thiazolidinediones ======= - metabolized in liver - excreted in feces and urine Adverse effects of Thiazolidinediones ======= - headache - sore throat - myalgia - upper resp infections - retention of fluid - raises HDL, TG, LDL Drug interactions of Thiazolidinediones ======= - use with insulin in heart failure patients - gemfibrozil (Lopid) - could have effect on liver so be careful with patients with liver problems - watch for dark coca-cola urine/jaundice etc.

Alpha Glucosidase Inhibitor ======= - Acarbose (Precose) - Uses Type 2 diabetes Action of Alpha Glucosidase Inhibitor ======= - delays the absorption of carbohydrates - reduces rise in blood sugar after meals - different from other insulins b/c action takes place in the gut Pharmacokinetics of Alpha Glucosidase Inhibitor ======= - little 2% absorbed Adverse effects of alpha Glucosidase Inhibitor ======= - flatulence - cramps - abdominal distention - diarrhea - can decrease iron absorption (watch for anemia) - hypoglycemia - liver dysfunction (acarbose) * mostly GI adverse effects Patient education with alpha Glucosidase Inhibitor ======= - must be taken with every meal Glyptins ======= - Sitagliptin (Januvia) - Uses = Type 2 diabetes Action of Glyptins ======= - enhance action of incretin hormones * incretin hormones are responsible for stimulating insulin secretion by beta cells incretin hormones ======= - glucagon like peptide 1 - glucose dependent insulinotropic polyptide What do incretin hormones do? ======= - promotes the secretion of insulin - Glyptins inhibits the break down of these hormones, therefore they are in the bloodstream longer and continue to release insulin (This is the mechanism of action for Glyptins) Pharmacokinetics of Glyptins ======= - renal excretion Adverse effects of Glyptins ======= - upper respiratory tract infections - inflammation of nasal passages and throat New drugs ======= 1. Amylin Mimetics 2. Incretin Mimetics Amilyn Mimetic ======= - Pramilinitide (Symlin) - given sub-q - decreases post-pyrandial glucose levels - delays gastric empyting, suppreses glucagon secretion, increases satiety - Enhancement to mealtime insulin - Adverse effects: hypoglycemia, nausea

Incretin mimetics ======= Exenatide (Byetta)- given Sub-Q - in combo with sulfonylurea and/or metformin What is one thing that sulfonylureas, glinides, glitazones, alpha-glucosidase inhibitors, biguanides gliptins have in common? ======= used for Type 2 diabetes Which oral antidiabetics drugs act by decreasing insulin resistance? ======= metformin - Sulfonylureas ... Which oral antidiabetics act by increasing insulin production? ======= - Sulfonylureas - Glynides Acute complications ======= - DKA - HYPERglycemic, HYPERosmolar non-ketotic syndrome - HYPOglycemia Diabetic Ketoacidosis Patho ======= - glucose metabolism --> hyperglycemia --> glycosuria --> osmotic diuresis --> water loss --> dehydration --> hemoconcentration (serum osmo is increased) - ketone production exceeds cellular use and renal excretion (break down proteins and fats to ketones) - Fat metabolism --> increased lipolysis --> increased FFA --> FFA oxidation --> production of ketoacids --> ketosis --> ketoacidosis signs and symptoms of diabetic ketoacidosis ======= - hyperglycemia (BG > 250) - production of ketoacids - Hemoconcentration (high HCT, Hgb, Osmo) - Acidosis (pH 6.8-7.3; HCO3 < 15) - coma - Kussmaul respirations - Fruity acetone odor to breath Treatment of DKA ======= - restore insulin levels (IV insulin) - correction of acidosis - Replace water and sodium with IV fluids (0.9 or 0.45% NS) - Normalize glucose - Correct electrolyte abnormalities (oral or IV K+ replacement) How is insulin given to adults? ======= - bolus dose followed by continuous insulin IV infusion - regular insulin normally - 0.1 - 0.15 units/kg - given IV instead of SC because of rapid acting describe rate of insulin infusion? ======= - 0.1 units/kg per hour - decreased when blood glucose is below 250 hang D5W and decrease bolus - when blood glucose is below 200 then titrate insulin

What happens if too much insulin is given and hypoglycemia occurs ======= - give glucose (D50) - or glucagon, but it takes longer Administration of K+ in DKA ======= - plasma K+ may be normal, but intracellular K+ may be low - insulin will cause K+ to shift into the cell and then you have HYPOkalemia - if plasma K+ is normal no supplement - if plasma K+ is low continue to supplement Why did your client with DKA present with heavy breathing or Kussmaul respirations? ======= Trying to blow off acid and compensate Hyperglycemic Hyperosmolar non-ketotic syndrome S&S ======= - HYPERglycemia (BG > 600) - no ketones - normal pH - hemoconcentration of HCT, Hgb, osmo hihg - electrolytes Hyperglycemic Hyperosmolar non-ketotic syndrome treatment ======= - fluid replacement - Correction of electrolyte disturbances - Administration of insulin Which of these symptoms would you not expect to see in a client with hyperglycemia without ketoacidosis? -heavy breathing? -Polyuria? -Dehydration? ======= Will not see: Heavy breathing Meds to know -insulin - biguanides - metformin (glucophage) - sulfonylureas- glipizide (glucotrol) - Glinides - repaglinide (prandin) - Glitazones - rosiglitazone (Avandia) - Alpha glucosidase inhibitors - Acarbose (precose) - Gliptins - sitagliptin (Januvia)

Following a meal, glucose that is not needed for immediate energy needs is stored as: Glycogen. o The storage from of glucose is called glycogen. Glucagon increases blood glucose level. The following body tissues or organs cannot synthesize glucose for its energy supply? o The brain The brain cannot because it is unable to synthesize glucose, and it will exhaust its supply after just a few minutes of activity.

The normal range for serum glucose is 60100 mg/dL. The level is usually tightly regulated to: o 8090 mg/dL o The body attempts to maintain tight glucose control to remain between 80 90 mg/dL to prevent complications associated by hypo- or hyperglycemic states.

The breakdown of fatty acids for fuel results in the presence of which of the following in the blood? B) Ketones B) When the body must metabolize fatty acids for fuel, ketones accumulate in the blood.

5) The primary factor contributing to the development of diabetes type 2 is: C) Obesity. C) 80% of persons with type 2 are overweight.

6) Risk factors for type 2 diabetes include which of the following? (Select all that apply.) A) Family history B) Obesity C) Race

D) Ethnicity A) Family history is a risk factor for the development of type 2 diabetes. B) Obesity is a risk factor for the development of type 2 diabetes. C) Race history is a risk factor for the development of type 2 diabetes. D) Ethnicity is a risk factor for the development of type 2 diabetes.

7) Polyuria that occurs in diabetes is due to: A) Osmotic diuresis. B) Excessive fluid intake. C) Inflammation of the glomerulus. D) Lack of albumin. Answer: A Explanation: A) Osmotic diuresis leads to polyuria, which is the passage of large amounts of urine as a result of increased osmotic pressure that can result from hyperglycemia. B) Excessive fluid intake does not lead to polyuria; it is due to osmotic diuresis. C) Inflammation of the glomerulus does not lead to polyuria; it is due to osmotic diuresis. D) Lack of albumin does not lead to polyuria; it is due to osmotic diuresis.

8) Hemoglobin A1C monitors: A) The level of hemoglobin over time. B) The level of glucose over time. C) The percent of glucose present in the blood. D) The percent of glucagon in the blood. Answer: B Explanation: A) Hemoglobin A1C measures the level of blood glucose over time because glucose molecules attach to the hemoglobin molecule for the life of the RBC, which is 120 days.

B) Hemoglobin A1C measures the level of blood glucose over time because glucose molecules attach to the hemoglobin molecule for the life of the RBC, which is 120 days. C) Hemoglobin A1C measures the level of blood glucose over time, not the presence of glucose in the blood, because glucose molecules attach to the hemoglobin molecule for the life of the RBC, which is 120 days. D) Hemoglobin A1C measures the level of blood glucose over time, not the present percent of glucagon in the blood, because glucose molecules attach to the hemoglobin molecule for the life of the RBC, which is 120 days.

9) The nurse assesses the client in the Emergency Department and finds the client confused, sweating, and complaining of a headache. The nurse suspects hypoglycemia. The nurse's next action is to obtain: A) A glucose tolerance test. B) A serum blood glucose. C) A finger-stick blood glucose. D) Serum electrolytes. Answer: C Explanation: A) Although each of the answer options is important, a glucose tolerance test measures glucose levels after two hours, and would not provide the immediate information needed to correct the hypoglycemia. B) Although each of the answer options is important, a serum blood glucose test would not provide the immediate information needed to correct the hypoglycemia. C) Although each of the answer options is important, obtaining a finger-stick blood sugar takes 15 seconds, and provides an opportunity for rapid nursing intervention, if needed. D) Although each of the answer options is important, serum electrolytes will not provide the level of blood sugar needed to intervene with hypoglycemia.

10) Which of the following drugs would alter the patient's ability to recognize the symptoms of hypoglycemia? A) Antibiotics B) Diuretics C) Beta blockers D) Oral hypoglycemics Answer: A Explanation: A) Beta blockers interfere with the symptoms of hypoglycemia, making it more difficult for the patient to recognize symptoms. Antibiotics do not cause this. B) Beta blockers interfere with the symptoms of hypoglycemia, making it more difficult for the patient to recognize symptoms. Diuretics do not cause this. C) Beta blockers interfere with the symptoms of hypoglycemia, making it more difficult for the patient to recognize symptoms. D) Beta blockers interfere with the symptoms of hypoglycemia, making it more difficult for the patient to recognize symptoms. Oral hypoglycemics do not cause this

11) The difference between diabetic ketoacidosis (DKA) and hyperosmolar hyperglycemic state (HHS) is that clients with HHS: A) Have elevated glucose. B) Are dehydrated. C) Have absence of ketoacidosis. D) Are confused. Answer: C Explanation: A) Patients with HHS have elevated glucose, but do not experience ketoacidosis.

B) Patients with HHS are dehydrated, but do not experience ketoacidosis. C) Patients with HHS do not experience ketoacidosis. D) Patients with HHS are confused; they do not experience ketoacidosis.

12) The leading cause of blindness in the United States is: A) Glaucoma. B) Retinopathy. C) Cataracts. D) Hypertension. Answer: B Explanation: A) The leading cause of blindness in the United States is retinopathy, not glaucoma. B) The leading cause of blindness in the United States is retinopathy due to microvascular changes associated with the hyperglycemic states of diabetes. C) The leading cause of blindness in the United States is retinopathy, not cataracts. D) The leading cause of blindness in the United States is retinopathy, not hypertension.

13) Which of the following should be included in patient teaching for the client with diabetes? (Select all that apply.) A) Maintain glucose control. B) Check feet daily for signs of irritation. C) Get annual eye exams. D) Monitor blood pressure carefully. E) Quit smoking. Answer: A, B, C, D, E Explanation: A) Maintaining glucose control is an important teaching point because tight glucose control helps prevent the complications of diabetes.

B) Checking the feet for signs of skin breakdown is an important teaching point because due to peripheral neuropathy, the client might not sense small changes in skin integrity in the feet. C) Getting annual eye exams is an important teaching point to facilitate early diagnosis of any microvascular changes in the eyes. D) Monitoring blood pressure carefully is an important teaching point to aid recognition of hypertensive changes associated with diabetes. E) Quitting smoking in an important teaching point because nicotine is a vasoconstrictor, and clients with diabetes are at greater risk for microvascular changes.

14) What percent of adults with diabetes take oral agents only? A) 3040 B) 4050 C) 5060 D) 6070 Answer: C Explanation: A) 57% of adults with diabetes take oral agents only. B) 57% of adults with diabetes take oral agents only. C) 57% of adults with diabetes take oral agents only. D) 57% of adults with diabetes take oral agents only.

15) The insulin with the most rapid onset of action is insulin: B) Lispro. The onset of lispro is 515 minutes. 16) Which insulin can be administered intravenously? A) Aspart B) Lispro C) Regular D) Glulisine

Answer: C Explanation: A) Only regular insulin can be administered IV; any other form would precipitate in solution. B) Only regular insulin can be administered IV; any other form would precipitate in solution. C) Only regular insulin can be administered IV; any other form would precipitate in solution. D) Only regular insulin can be administered IV; any other form would precipitate in solution.

17) The primary adverse effect of insulin is: A) Somogyi phenomenon. B) Swollen lymph glands. C) Urticaria. D) Hypoglycemia. Answer: D Explanation: A) Somogyi phenomenon is a result of hormonal change. Hyperglycemia can occur, but the primary adverse effect of insulin is hypoglycemia. B) Swollen lymph glands can occur but, the primary adverse effect of insulin is hypoglycemia. C) Urticariaitching hivescan occur, but the primary adverse effect of insulin is hypoglycemia. D) The primary adverse effect of insulin is hypoglycemia.

18) The advantage of second-generation sulfonylureas is they: A) Exhibit fewer drugdrug interactions. B) Can be administered in smaller doses. C) Cause fewer side effects. D) Do not cause hypoglycemia. Answer: A

A) The advantage of sulfonylureas is that they exhibit fewer drugdrug interactions. B) Smaller doses are not an advantage for sulfonylureas. The advantage is they cause fewer drugdrug interactions. C) The sulfonylureas do not cause fewer side effects. Their advantage is they cause fewer drugdrug interactions. D) The sulfonylureas cause hypoglycemia. The advantage is they cause fewer drug drug interactions.

19) Which of the following substances should be avoided while the client is using sulfonylureas? A) Calcium products B) Alcohol C) Antacids D) Antibiotics Answer: B A) When administered with alcohol, sulfonylureas can cause a disulfiram-like reaction with flushing, palpations, and nausea. B) When administered with alcohol, sulfonylureas can cause a disulfiram-like reaction with flushing, palpations, and nausea. C) When administered with alcohol, sulfonylureas can cause a disulfiram-like reaction with flushing, palpations, and nausea. D) When administered with alcohol, sulfonylureas can cause a disulfiram-like reaction with flushing, palpations, and nausea.

20) Glyburide would be contraindicated in patient with allergies to: A) Sulfa drugs. B) Eggs.

C) Milk products. D) Urea. A) Glyburide is a sulfonylurea, and is contraindicated in patients with a known sensitivity to sulfa drugs. B) Glyburide is a sulfonylurea, and is contraindicated in patients with a known sensitivity to sulfa drugs. Eggs are not a factor C) Glyburide is a sulfonylurea, and is contraindicated in patients with a known sensitivity to sulfa drugs. Milk products are not a factor. D) Glyburide is a sulfonylurea, and is contraindicated in patients with a known sensitivity to sulfa drugs. Urea is not a factor

21) The nurse knows that insulin should not be administered in patients whose blood glucose is lower than: A) 90 mg/dL. B) 80 mg/dL. C) 70 mg/dL. D) 60 mg/dL. Answer: C A) The nurse should not administer insulin if blood glucose levels are lower than 70 mg/dL. B) The nurse should not administer insulin if blood glucose levels are lower than 70 mg/dL. C) The nurse should not administer insulin if blood glucose levels are lower than 70 mg/dL. D) The nurse should not administer insulin if blood glucose levels are lower than 70 mg/dL.

22) Insulin administration site should be rotated primarily to prevent: A) Lipodystrophy. B) Systemic absorption of insulin. C) Ineffective dosing.

D) Abscess development. Answer: A A) Rotation of insulin sites helps to prevent lipodystrophy. B) Insulin rotation would not prevent systemic absorption. C) Insulin site rotation does not prevent infective dosing. D) Rotation of sites could help prevent abscess development, but this is not the primary.

23) Which of the following should be included in the patient teaching plan for the client taking insulin? (Select all that apply.) A) Rotate sites. B) Carry a readily available supply of sugar. C) Remember the signs of hypoglycemia. D) Wear a medic alert bracelet that explains the patient has diabetes. E) Store insulin in the freezer. Answer: A, B, C, D A) Insulin sites should be rotated to prevent lipodystrophy. B) Diabetes should carry a readily available source of sugar. C) Patients should understand the symptoms of hypo- and hyperglycemia. D) Patient should wear a medic alert bracelet identifying them as diabetics. E) Insulin should be stored in the refrigerator, not the freezer.

Hyperglycemia -------- High blood glucose levels Polyuria -------- Frequent and excessive urination. Results from an osmotic diuresis caused by excess glucose in the urine. Hyperosmalarity of the extracellular fluids secondary to hyperglcemia Sodium, chloride and potassium are excreted. Polydipsia -------- Excessive thirst Polypagia -------- Excessive eating triggered by cell starvation Dehydration -------- Leads to: 1. Hemoconcentration: and increased blood concentration 2. Hypovolemia: a decreased blood volume 3. Hyperviscosity: thick concentrated blood 4. Hypoperfusion: decreased circulation 5. Hypoxia: poor tissue oxygenation Metabolic Acidosis -------- Excess acids caused by absence of insulin, increase hydrogen ion (H+) and carbon dioxide (CO2) levels in the blood. Kussmaul Respiration -------- Metabolic acidosis triggers the respiratory centers of the brain to increase the rate and depth of respiration in an attempt to excrete more carbon dioxide and acid. When lungs can no longer offset acidosis, blood pH drops. ABG -------- Show a metabolic acidosis (decreased pH with decreased arterial bicarbonate (HCO3) levels) and compensatory respiratory alkalosis (decreased partial pressure of arterial carbon dioxide (Paco2)) Hypekalemia -------- low serum potassium levels hyperkalemia -------- high serum potassium levels Acute Complications of Diabetes -------1. Diabetic ketoacidosis 2. Hyperglycemic - hyperosmolar-non ketotic syndrome 3. hypoglycemia Diabetic Ketoacidosis -------- Caused by lack of insulin and ketosis: increased depth/rate of respirations

increased urine output tachycardia orthostatic hypotension Hyperglycemic - hyperosmolar state -------- Caused by insulin deficiency and profound dehydration Hypoglycemia -------- from too much insulin or too little glucose Chronic Complications of Diabetes -------- Cardiovascular Disease Cerebrovascular Disease Retinopathy (vision) Problems Diabetic Neuropathy Diabetic Nephropathy Male erectile dysfunction Cardiovasular Disease -------- Most common complication of diabetes mellitus ADA recommends to maintain: BP below 130/80 LDL cholesterol be lowerd to less than 100mg/dl for patients without manifestations of CVD and less than 70mg/dl for pts with manifestation of CVD Cerebrovascular Disease -------- Diabetes damages cerebrovascular (brain) arterial circulation and is a risk factor for stroke Retinopathy problems -------- Legal blindness is 25 times more common in patients with diabetes. Proliferateive diabetic retinopahty (PDR) is the growth of new retinal blood vessels, also known as neovascularization. Diabetic Neuropathy -------- a progressive deterioration of nerves that results in loss of nerve function. Diabetic Nephropathy -------- a pathologic change in the kidney that reduces kidney function and leads to kidney failure. Male erectile dysfunction -------- the inability to achieve or maintain a penile erection sufficient for satisfactory sexual performance. Type 1 -------- an autoimmune disorder in immune system cells, mediators, and antibodies attack and destroy insulin-secreting cells (beta) in the islets. Type 1 S/S -------- Polyuria - frequent and excessive urination Polydipsia - excessive thirst Polyphagia - excessive eating Weight loss Weakness Type 2 -------- a progressive disorder in which the pancreas makes less insulin over time.

Type 2 S/S -------- Polyuria - frequent and excessive urination Polydipsia - excessive thirst Polyphagia - excessive eating Weight loss Weakness Visual blurring Vascular or neural complications Type 2 - Metabolic syndrome -------- also called syndrome X, is the simultaneous presence of metabolic factors known to increase risk for developing type 2 diabetes and cardiovascular disease. Abdominal obesity Hyperglycemia Hypertension Dyslipidemia Dyslipidemia -------- triglyceride level 150mg/dl or more or on drug treatment for elevated triglycerides; high-density lipoprotien (HDL) cholesterol less than 40mg/dl for men or less than 50mg/dl for women. Other types of diabetes -------- Gestational Genetic defect beta cell or insulin Disease of exocrine pancreas Drug or chemical induced Infections Other gentic syndromes Assessment -------- History Blood test: to diagnose diabetes Fasting blood glucose test - 2 test >126 mg/dl Oral Glucose tolerance test - blood glucose > 200 mg/dl at 120 minutes Glycosylated hemoglobin assays (HbAiC, A1C) - >8% Glucosylated serum protiens and albumin Urine Tests -------- The ADA recommends testing uringe for ketones during acute illness or stress, when blood glucose levels consistently exceed 300 mg/dl during pregnancy, or when any symptoms of ketoacidosis are present. Ketone testing is recommended for diabetic clients following a weight loss program Urine tesing for renal function Urine testing for glucose Nursing Diagnosis -------- Risk for injury, Related to Hyperglycemia S/S - hot, dry skin; dehydrated, rapid deep Kussmaul respirations with frutiy odor, coma, acidosis, abdominal cramps, N/V, orthostatic hypotension, tachycardia, poor skin turger, glucose > 250. Positive ketones. Interventions -------- Dietary interventions, blood glucose monitoring, exercise program, weight control

Oral Drug Therapy -------- Sulfonylurea agents: are classified as insulin secretagogues and are used for patients with some remaining pancreatic beta-cell functions. (ie Diabinese, Tolinase, Glucotrol, Glyburide (Micronase), Amarly) Meglitinide analogues: are classified as insulin secretagogues and have actions and adverse effects similar to those of sulfonylureas. (ie Pradin) Oral Drug Therapy -------- Biguanides: are antihyperglycemic agents and insulin sensitizers (ie Metformin (Glucophage)) Alpha- glucosidase inhibitors: are agents that prevent hyperglycemia by delaying absorption of carbohydrate from the small intestine. (ie Arcarbose (Precose)) Oral Drug Therapy -------- Thiazolinedione antidiabetic agents: are antihyperglycemic agents and insulin sensitizers, they improve insulin sensitivity and reduce liver glucose production. (ie Avandia, Actos) Combination agents: combines drugs with different michanisms of action. (ie Glucovance) Drug administration -------- Drugs are started at the lowest effective dose and increased every 1 to 2 weeks until the pt reaches desired blood glucose control or the maximun dosage Drug selection -------- The choice of oral antidiabetic drug is based on cost, the patient's ability to manage multiple drug doses, age and response to the drugs. Insulin Therapy -------- Rapid Acting - Humalog; Novolog Short-acting insulin (Regular) - Humulin R, Novolin R (Regular Insulin) Intermediate - Humulin N; Novolin NPH Combinations: NPH/Regular Long Acting - Glargine (Lantus) Insulin Regimens -------- Single daily injection protocol - require insulin injection only once a day, could be intermediate or long-acting or a combination of both Two-dose protocol - combines short and intermediate- acting insulin injected twice daily. 2/3 given before breakfast and 1/3 before the evening meal Insulin Regimens -------- Three-dose protocol Four-dose protocol Combination therapy Intensified therapy regimens - include a basal dose of intermediate or long-acting insulin and a bolus dose of short or rapid-acting insulin designed to bring the next blood glucose value into the target range. Pharmacokinetics of Insulin -------- Injection site - affects the speed of absorption, absorption is faster in the abdomen, rotation prevents lipohypertrophy

Absorption rate - is determined by insulin properties, the longer the duration of action the more unpredictable absorption is. Injection depth - changes the insulin absorption, 90-degree angle, aspiration is not needed Pharmacokinetics of Insulin -------- Time of injection - affects blood glucose levels, between premeal injections and eating, kown as "lag time" affects blood glucose levels after meals Mixing insulins - can change the time of peak action, mixtures of short and intermediateacting insulins produce a more normal blood glucose responce in some patients thatn does a single dose. Insulin -------- When rapid-acting (Humalog or NovoLog) or short-acting (regular) insulin is mixed with a longer-acting insulin, draw the shorter-acting dose into the syringe first. Complications of Insulin Therapy -------- Hypoglycemia Lipoatrophy - is a loss of fat tissue in areas of repeated injection that results from an immune reaction to impurities in insulin Dawn phenomenon - results from a nighttime release of growth hormone that causes blood glucose elevations at about 5 to 6 am Somagyi's phenomenon - is morning hyperglycemia from the counterregulatory response to nighttime hypoglycemia Alternative Methods of Insulin Administration -------- Continuous subcutaneous infusion of insulin - of a basal dose of insulin (CSII) with increases in insulin at mealtimes ismore effective in controlling blood glucose levels than a multiple-injection schedule Implanted insulin pumps Injection devices Chapter 67 -------- Interventions for Clients with Diabetes Mellitus Absence of Insulin -------- Hyperglycemia Polyuria hyperosmolarity Polydipsia Polyphagia Dehydration: Hemoconcentration, hypovolemia, hyperviscosity, hypoperfusion, and hypoxia Acidosis, Kussmaul respiration ABG's: DECREASED pH, HCO3, and PaCO2 Hypokalemia, hyperkalemia, or normal serum potassium levels Diabetic ketoacidosis -------- Increased depth/rate of respirations Increased urine output Tachycardia Orthostatic hypotension

Hyperglycemic-hyperosmolar-nonketotic syndrome -------- Hyperglycemia, hyperosmolarity, dehydration, absence of ketosis, depression of sensorium Seen with Type 2 DM Hypoglycemia -------- from too much insulin or too little glucose Cardiovascular disease -------- Managed (ADA) by: BP at 130/80; LDL<100mg/dl(pts with CVD) and <70 for those without CVD);Retinopathy (vision) problems Diabetic neuropathy Diabetic nephropathy Male erectile dysfunction Types of Diabetes -------- type 1 and type 2 Type I -------- S/S: 3 P's; weight loss, weakness Type 2 -------- S/S: Type 1 + visual blurring vascular or neural complications Metabolic Syndrome -------- Abdominal obesity Hyperglycemia HPTN Dyslipidemia Gestational Diabetes -------- First appears in pregnancy and disappears after pregnancy is terminated Other types include: -------- Genetic defect beta cell or insulin Disease of exocrine pancreas Drug or chemical induced Infections Other genetic syndromes. Assessment -------- History Blood tests: to diagnose diabetes Fasting blood glucose test: two tests > 126 mg/dL Oral glucose tolerance test: blood glucose > 200 mg/dL at 120 minutes Glycosylated hemoglobin assays (HbAiC, A1C) >8% Glucosylated serum proteins and albumin Urine Tests -------- The ADA recommends testing urine for ketones during acute illness or stress, when blood glucose levels consistently exceed 300 mg/dL (16.7 mmol/L), during pregnancy, or when any symptoms of ketoacidosis are present (ADA, 2003s). Ketone testing is recommended for diabetic clients following a weight loss program Urine testing for renal function Urine testing for glucose

Risk for Injury Related to Hyperglycemia -------- hot,dry skin; dehydrated, rapid deep Kussmaul typye with fruity order respirations, MS is alert to coma; acidosic, abdominal cramps, N/V, orthostatic hypotension, tachycardia, poor skin tuger, glucose > 250. positive ketones. Interventions include: -------- Dietary interventions, blood glucose monitoring, exercise program, weight control. Dietary interventions -------- blood glucose monitoring, exercise program, weight control. Sulfonylurea agents -------- Diabinese Tolinase Glucotrol Glyburide (Micronase) Amaryl Meglitinide analogues -------- Prandin Biguanides -------- Metformin (Glucophage) Alpha-glucosidase inhibitors -------- Acarbose (Precose) Thiazolinedione antidiabetic agents -------- Avandia Actos Combination Agents -------- Glucovance Insulin therapy: -------- Insulin therapy: Rapid Acting Humalog; Novolog Short-acting insulin (Regular) Humulin R, Novolin R (Regular Insulin) Intermediate Humulin N; Novolin NPH Combinations: NPH/Regular Long Acting Glargine (Lantus) Insulin Regimens -------- Single daily injection protocol Two-dose protocol Three-dose protocol Four-dose protocol Combination therapy Intensified therapy regimens Pharmacokinetics of Insulin -------- Injection site Absorption rate Injection depth Time of injection Mixing insulins

Complications of Insulin Therapy -------- Hypoglycemia Lipoatrophy Dawn phenomenon Somagyi's phenomenon Dawn phenomenon -------- Early morning glucose elevation produced by the release of growth hormone, which decreases peripheral uptake of glucose resulting in elevated morning glucose levels. Admin of insulin at a later time in day will coordinate insulin peak with the hormone release. Alternative Methods of Insulin Administration -------- Continuous subcutaneous infusion of insulin Implanted insulin pumps Injection devices Client Education -------- Storage and dose preparation Syringes Blood glucose monitoring Interpretation of results Frequency of testing Blood glucose therapy goals Diet Therapy -------- Goals of diet therapy Principles of nutrition in diabetes Protein, fats and carbohydrates, fiber, sweeteners, fat replacers Alcohol Food labeling Exchange system, carbohydrate counting Special considerations for type 1 and type 2 diabetes Exercise Therapy -------- Benefits of exercise Risks related to exercise Screening before starting exercise program Guidelines for exercise Exercise promotion Interventions and foot care practices: -------- Cleanse and inspect the feet daily. Wear properly fitting shoes. Avoid walking barefoot. Trim toenails properly. Report nonhealing breaks in the skin. Wound Care -------- Wound environment Debridement Elimination of pressure on infected area Growth factors applied to wounds Chronic Pain -------- Maintenance of normal blood glucose levels

Anticonvulsants Antidepressants Capsaicin cream Risk for Injury Related to Disturbed Sensory Perception: Visual -------- Interventions include: Blood glucose control Environmental management Incandescent lamp Coding objects Syringes with magnifiers Use of adaptive devices Ineffective Tissue Perfusion: Renal -------- Interventions include: Control of blood glucose levels Yearly evaluation of kidney function Control of blood pressure levels Prompt treatment of UTIs Avoidance of nephrotoxic drugs Diet therapy Fluid and electrolyte management Potential for Hypoglycemia -------- Blood glucose level < 70 mg/dL Diet therapy: carbohydrate replacement Drug therapy: glucagon, 50% dextrose, diazoxide, octreotide Prevention strategies for: Insulin excess Deficient food intake Exercise Alcohol Pg 1508: chart 67-11 Potential for Diabetic Ketoacidosis -------- Interventions include: Monitoring for manifestations Assessment of airway, level of consciousness, hydration status, blood glucose level Management of fluid and electrolytes Drug therapy goal: to lower serum glucose by 75 to 150 mg/dL/hr Management of acidosis Client education and prevention Potential for Hyperglycemic-Hyperosmolar Nonketotic Syndrome and Coma -------Interventions include: Monitoring Fluid therapy: to rehydrate the client and restore normal blood glucose levels within 36 to 72 hr Continuing therapy with IV regular insulin at 10 units/hr often needed to reduce blood glucose levels Health Teaching -------- Assessing learning needs Assessing physical, cognitive, and emotional limitations Explaining survival skills

Counseling Psychosocial preparation Home care management Health care resources Pg 1518: NCLEX pointers 1514 hyperosmolarity -------- of the extracellular fluids secondary to hyperglycemia, more solute in relation to the solvent. Hyperglycemia -------- abnormally high blood sugar usually associated with diabetes Polyuria -------- renal disorder characterized by the production of large volumes of pale dilute urine Polydipsia -------- excessive thirst (as in cases of diabetes or kidney dysfunction) Polyphagia -------- excessive hunger and eating Dehydration -------- a serious reduction in the body's water content Hemoconcentration -------- A decrease in the fluid content of the blood (plasma), resulting in an increase concentration. This is determined by an increase in the hematocrit. Caused by a filtration of plasma into the body tissue and often created by dehydration hypovolemia -------- a blood disorder consisting of a decrease in the volume of circulating blood hyperviscosity -------- Waldenstrom's Macroglobulinemia (impairs blood flow, sludging in retinval vessels and cerebral vasculatrue) hypoperfusion -------- a deficiency of blood passing through an organ or body part hypoxia -------- lack of oxygen diabetes mellitus -------- endocrine disorder of the pancreas that affects carbohydrate, fat and protein metabolism diabetic ketoacidosis -------- type of metabolic acidosis that occurs when there is acute insulin deficiency diabetic nephropathy -------- progressive decrease in renal function that occurs with diabetes mellitus diabetic retinopathy -------- pathologic changes in the retina experienced by persons with diabetes fasting blood glucose -------- blood test performed to detect and monitor diabetes 70 - 110 mg/dL

glycemic index -------- measure of how fast a carbohydrate food is likely to raise blood sugar glucosuria -------- glucose in the urine glycosylated hemoglobin (A1C) -------- amount of glucose stored with a hemoglobin molecule during its lifespan of 120 days glycogenolysis -------- process in which glycogen is broken down into glucose in the liver hyperglycemia -------- elevated blood glucose level hyperosmolar hyperglycemic nonketotic syndrome (HHNKS) -------- acute complication of diabetes characterized by hyperglycemia without ketosis hypoglycemia -------- low blood glucose level insulin independence -------- ability of a client's naturally produced insulin to regulate blood glucose levels within consistently normal ranges insulin -------- pancreatic hormone necessary for the metabolism of glucose insulin resistance -------- decreased sensitivity to insulin at the tissue level ketoacidosis -------- form of metabolic acidosis resutting from an accumulation of ketones in the blood ketones -------- metabolic by product of fat metabolism ketonemia -------- increased ketones in the blood Kussmaul respirations -------- fast, deep breathing lipoatrophy -------- breakdown of subcutaneous fat at the site of repeated injections lipohypertrophy -------- buildup of subcutaneous fat at the site of repeated injections lipolysis -------- breaking down of fat by the body metabolic syndrome -------- cluster of physiologic alterations that include obesity, HTN, elevated tryglycerides, LDL, BS and low HDL oral glucose tolerance test -------- blood test used to evaluate a client's metabolism of orally ingested glucose polydipsia -------- excessive thirst polyphagia -------- excessive hunger polyuria -------- excessive urine production

post prandial glucose -------- blood test used to assess blood sugar following a meal in non-diabetic 70 - 110 mg/dL pre-diabetes -------- condition characterized by impaired fasting glucose (100-125 mg/dL), impaired glucose tolerance (140-199 mg/dL after 2 hours) or both random blood glucose -------- blood specimen drawn without preplanning >200 mg/dL in presence of symptoms is suggestive of diabetes renal threshold -------- ability of the kidney to absorb glucose and return it to the blood stream rule of 15 -------- give 15 g of rapidly absorbed CHO, wait 15 minutes then rechecck blood sugar, give another 15 g of CHO if BS is not above 70 mg/dL tight glucose control -------- maintaining near normal blood glucose levels by taking shortacting insulin throughout the day and intermediate acting insulin at bedtime Type I diabetes -------- characterized by no insulin production by the pancreas Type 2 diabetes -------- characterized by insulin resistance or insufficient insulin production Insulin Lispro (Humalog) -------- onset 5-15 minutes peak 1-2 hours duration 3-4 hours Aspart (Novolog) -------- onset 15 minutes peak 1 hour duration 3-4 hours Regular Insulin (Humulin R, Novolin R) -------- onset 30 - 60 minutes peak 1 - 3 hours duration 6 - 8 hours NPH, Humulin N, Novolin N -------- onset 1 - 1.5 hours peak 4-12 hours duration 24 hours Lente -------- onset 1 - 2.5 hours peak 7 - 15 hours duration 24 hours Ultralente, Humulin U -------- onset 4 - 8 hours peak 8 - 10 hours duration 18 - 30 hours Glargine (lantus) -------- onset 2 - 4 hours peak NO PEAK duration >24 hours

Humulin 50/50 -------- onset 15 minutes peak 2 - 4 hours duration 22-22 hours Humulin 70/30 -------- onset 30 minutes peak 7 - 12 hours duration 16 - 24 hours Novolin 70/30 -------- onset 30 minutes peak 3 - 8 hours duration 10 - 16 hours Humalog 75/25 -------- onset 15 minutes peak 2 hours duration 20 - 22 hours Name 8 categories of DM drugs. -------Insulin Sulfonylureas Biguanides Alpha-glucosidase inhibitors Glitazones/Thiazolidinediones Meglitinides DPP-4 inhibitor Mimetics Name 6 types of insulin. -------Lispro Aspart Regular NPH Glargine Detemir Which insulins are rapid-acting? -------Lispro Aspart Regular Which insulin is intermediate acting? -------- NPH Which insulins are long-acting? -------Glargine Detemir What is the main possible SE of insulin? What about a rare SE? -------Main = Hypoglycemia Rare = Hypersensitivity rxn What is the mechanism of action of insulin? -------- It binds insulin receptor (tyrosine kinase activity)

Describe insulin's affect on the liver, muscle, and fat. -------Liver- inc. storage of glucose as glycogen Muscle- inc. glycogen and protein syn., inc. K uptake Fat- inc. TG storage Name 5 conditions in which insulin could be indicated? -------Type I DM Type 2 DM Gestational diabetes Life-threatening hyperkalemia Stress-induced hyperglycemia Name 5 types of sulfonylureas. -------First generation: Tolbutamide Chlorpropamide Second generation: Glyburide Glimepiride Glipizide What is the mechanism of action of sulfonylureas? -------- They increase production of insulin from the pancreas. Mech. of action: close K channels in beta-cell membrane -> cell depolarizes -> Ca influx -> triggers insulin release Name 1 SE effect of first generation sulfonylureas and 1 SE of second generation sulfonylureas. -------- First generation: GI (nausea, vomiting, heartburn, etc.) Second generation: Hypoglycemia Name the indication for using sulfonylureas. -------- Type 2 DM Note: they require some islet function, so they are useless in type I DM Name 1 type of biguanides. -------- Metformin What is the mechanism of action of biguanides? -------- Decrease production of glucose from liver ( glycolysis and gluconeogenesis), AND improve insulin utilization in tissues (better insulin sensitivity) Mechanism of action: unknown Name the indications for using biguanides. -------- Type I DM (can be used w/o islet function) Type II DM Name 2 types of glitazones/thiazolidinediones. -------- Pioglitazone Rosiglitazone

What is the mechanism of action of glitazones/thiazolidinediones? -------- They inc. insulin sensitivity in peripheral tissue AND dec. hepatic glucose production. Mechanism of action: bind to PPAR-gamma nuclear transcription regulator Name the indication for glitazones/thiazolidinediones. -------- Type 2 DM Name 5 SE's of glitazones/thiazolidinediones. -------Weight gain Edema Hepatotoxicity CV toxicity N/V/D Name 2 types of alpha-glucosidase inhibitors. -------Acarbose Miglitol What is the mechanism of action of alpha-glucosidase inhibitors? -------- They inhibit intestinal brush-border alpha-glucosidases -> slow digestion/absorption of carbs -> postprandial glucose Name the indication for alpha-glucosidase inhibitors. -------- Type 2 DM Name 3 SE's of alpha-glucosidase inhibitors. -------GI: FLATULENCE Diarrhea Abdominal pain Name 2 types of mimetics. -------- Amylin mimetic Incretin mimetic Name an Amylin mimetic. -------- Pramlintide What is the mechanism of action of Amylin mimetics? -------Dec. postprandial glucose by: -Slowing gastric emptying -Dec. glucagon secretion (and hepatic glucose production) -Centrally-mediated appetite suppression What are 3 SE's of Amylin mimetics? -------Hypoglycemia N/D Anorexia Name an incretin mimetic. -------- Exenatide What is the mechanism of action of incretin mimetics? -------- They inc. insulin secretion and dec. glucagon secretion.

Mechanism of action: they are GLP-1 analogs What is the indication for mimetics? -------- Type 2 DM Name 2 types of Meglitinides. -------- Repaglinide Nateglinide What is the mechanism of action of meglitinides. -------- They inc. insulin secretion Name 4 SE's of meglitinides. -------- Hypoglycemia URI's N/V/D Abdominal pain Name 1 type of DPP-4 inhibitor. -------- Sitagliptin What is the mechanism of action of DPP-4 inhibitors? -------- They inc. glucose-dep. insulin secretion and dec. glucagon secretion. Mechanism of action: inhibit dipeptidyl-peptidase-4 What are 3 SE's of DPP-4 inhibitors. -------- URI's Nasopharyngitis HA Classifications of Insulin: -------- Rapid acting insulin, short acting insulin, intermediate acting insulin, long acting insulin, combination therapy (premixed). Rapid acting insulins: -------- lispro (Humalog), aspart (Novolog), glulisine (Apidra). Clarity of Rapid Acting Insulins: -------- Clear. Short acting insulins: -------- regular (Humulin R, Novolin N, ReliOn N). Clarity of Short Acting Insulins: -------- Clear. Intermediate Acting Insulins: -------- NPH (Humulin N, Novolin N, ReliOn N). Clarity of Intermediate Acting Insulins: -------- Cloudy. Long Acting Insulins: -------- glargine (Lantus), determir (Levemir). Clarity of Long Acting Insulins: -------- Clear. Combination Therapy Insulins (Premixed): -------- NPH/regular 70/30 (Humulin 70/30, Novolin 70/30, ReliOn 70/30), NPH/regular 50/50 (Humulin 50/50), lispro protamine/lispro 75/25 (Humalog Mix 75/25), lispro protamine/lispro 50/50 (Humalog mix 50/50), aspart protamine/aspart 70/30 (NovoLog 70/30).

Type of Insulin/Frequency of Once a day regimen dose: -------- Intermediate (NPH) at bedtime, or long acting (glargine Lantus, determir Levemir) in am or at bedtime. Twice a day regimen dose? -------- Split mixed dose. Type of Insulin/Frequency of Twice a Day Regimen dose: -------- NPH and regular or rapid, before breakfast and at dinner. Three times a day regimen dose: -------- Combination of mixed and single dose. Type of Insulin/Frequency of Three Times a Day Regimen dose: -------- NPH and regular or rapid before breakfast + regular or rapid before dinner + NPH at bedtime. Basal Bolus Regimen Dose: -------- Multiple dose. Type of Insulin/Frequency of Basal Bolus Regimen Dose: -------- Regular or rapid before breakfast, lunch and dinner + long acting once a day or regular or rapid before breakfast, lunch, and dinner + NPH twice a day. lispro (humalog) -------- rapid acting insulin lispro humalog -------- onset 10-15 minutes lispro humalog -------- peak 1 hour lispro humalog -------- duration 2-4 hours lispro humalog -------- used for rapid reduction of glucose level, to treat postprandial hyperglycemia, and/or to prevent nocturnal hypoglycemia aspart novolog -------- rapid acting insulin aspart novolog -------- onset 5-15 min aspart novolog -------- peak 40-50 min aspart novolog -------- duration 2-4 hours aspart novolog -------- used for rapid reduction of glucose level, to treat postprandial hyperglycemia, and/or to prevent nocturnal hypoglycemia glulisine apidra -------- rapid acting insulin glulisine apidra -------- onset 5-15 mins glulisine apidra -------- duration 2 hours glulisine apidra -------- used for rapid reduction of glucose level, to treat postprandial hyperglycemia, and/or to prevent nocturnal hypoglycemia short acting insulins -------- regular (Humalog R, Novolin R, Iletin II Regular)

regular (Humalog R, Novolin R, Iletin II Regular) -------- onset 1/2-1 hours regular (Humalog R, Novolin R, Iletin II Regular) -------- peak 2-3 h regular (Humalog R, Novolin R, Iletin II Regular) -------- duration 4-6 hours regular (Humalog R, Novolin R, Iletin II Regular) -------- usually administered 20-30 minutes before a meal regular (Humalog R, Novolin R, Iletin II Regular) -------- may be taken alone or in combination with longer-acting insulin Intermediate acting insulin -------- NPH NPH -------- onset 2-4 hours NPH -------- peak 4-12 hours NPH -------- duration 16-20 hour NPH -------- usually taken after food Humulin N, Iletin II Lente, Iletin II NPH, Novolin L (Lente), Novolin N (NPH) -------- onset 3-4 hours Humulin N, Iletin II Lente, Iletin II NPH, Novolin L (Lente), Novolin N (NPH) -------- peak 412 hours Humulin N, Iletin II Lente, Iletin II NPH, Novolin L (Lente), Novolin N (NPH) -------- duration 16-20 hours Humulin N, Iletin II Lente, Iletin II NPH, Novolin L (Lente), Novolin N (NPH) -------- usually taken after food very long acting insulins -------- Glargine (Lantus) Detemir (Levemir) Glargine (Lantus) Detemir (Levemir) -------- onset 1 hour Glargine (Lantus) Detemir (Levemir) -------- continuous (no peak) Glargine (Lantus) Detemir (Levemir) -------- duration 24 hours Glargine (Lantus) Detemir (Levemir) -------- used for basal dose of insulin 3 causes of morning hyperglycemia -------- insulin waning

dawn phenomenon somogyi effect insulin waning -------- progressive rise in blood glucose from bedtime to morning treatment for insulin waning -------- increase evening (predinner or bedtime) dose of intermediate acting or long acting insulin, or institute a dose of insulin before the evening meal if one is not already part of the treatment regimen dawn phenomenon -------- relatively normal blood glucose until about 3am, when the level begins to rise treatment for dawn phenomenon -------- change the time of injection of evening intermediate acting insulin from dinnertime to bedtime somogyi effect -------- normal or elevated blood glucose at bedtime, a decrease at 2-3am to hypoglycemic levels and a subsequent increase caused by the production of counterregulatory hormones treatment of somogyi effect -------- decrease evening (predinner or bedtime) dose of intermediate acting insulin, or increase bedtime snack first generation sulfonylureas -------- acetohexamide (Dymelor) chlorpropamide (Diabinese) tolazamide (tolinase) Tolbutamide (orinase) first generation sulfonylureas action/indications acetohexamide (Dymelor) chlorpropamide (Diabinese) tolazamide (tolinase) Tolbutamide (orinase) -------- action/indications *used in type 2 diabetes to control blood glucose levels *stimulates beta cells of the pancreas to secrete insulin *may improve binding between insulin and insulin receptors or increase the number of insulin receptors side effects of first generation sulfonylureas acetohexamide (Dymelor) chlorpropamide (Diabinese) tolazamide (tolinase) Tolbutamide (orinase) -------- SE *hypoglycemia *mild GI symptoms *weight gain *drug-drug interactions (NSAIDS, warfarin, sulfonamides) *sulfa allergy *skin reactions implications of first generation sulfonylureas acetohexamide (Dymelor)

chlorpropamide (Diabinese) tolazamide (tolinase) Tolbutamide (orinase) -------- implications *monitor pt for hypoglycemia *monitor blood glucose and urine ketone levels to assess effectiveness of therapy implications of first generation sulfonylureas acetohexamide (Dymelor) chlorpropamide (Diabinese) tolazamide (tolinase) Tolbutamide (orinase) -------- implications *patients at high risk for hypoglycemia: advanced age, renal insufficiency *When taken wih beta-adrenergic blocking agents, may mask usual warning signs and symptoms of hypoglycemia implications of first generation sulfonylureas acetohexamide (Dymelor) chlorpropamide (Diabinese) tolazamide (tolinase) Tolbutamide (orinase) -------- implications *instruct patients to avoid the use of alcohol *check for interactions with other meds second generation sulfonylureas -------- *Glipizide (glocotrol, glocotrol XL) *Glyburide (Micronase, Glynase, Dia-Beta) *Glimepiride (Amaryl) second generation sulfonylureas action/indications *Glipizide (glocotrol, glocotrol XL) *Glyburide (Micronase, Glynase, Dia-Beta) *Glimepiride (Amaryl) -------- action/indications *stimulates beta cells to secrete insulin *may improve binding between insulin and insulin receptors or increase the number of insulin receptors) *Used in type 2 *more potent effects than 1st gen *Glipizide (glocotrol, glocotrol XL) *Glyburide (Micronase, Glynase, Dia-Beta) *Glimepiride (Amaryl) action/indications -------- action/indications May be used in combination with metformin or insulin to improve glucose control *Glipizide (glocotrol, glocotrol XL) *Glyburide (Micronase, Glynase, Dia-Beta) *Glimepiride (Amaryl) 2nd gen sulfonylureas side effects -------- side effects *Hypoglycemia *mild GI symptoms *Weight gain

*Drug-drug interactions (NSAIDS, warfarin, sulfonamides) *sulfa allergies *Glipizide (glocotrol, glocotrol XL) *Glyburide (Micronase, Glynase, Dia-Beta) *Glimepiride (Amaryl) 2nd gen sulfonylureas implications -------- implications *monitor pt for hypoglycemia *monitor blood glucose and urine ketone levels to assess effectiveness of therapy *patients at high risk for hypoglycemia: advanced age, renal insufficiency implications *Glipizide (glocotrol, glocotrol XL) *Glyburide (Micronase, Glynase, Dia-Beta) *Glimepiride (Amaryl) 2nd gen sulfonylureas -------- implications *when taken with beta blockers: may mask usual warning signs and symptoms of hypoglycemia *instruct patients to avoid the use of alcohol biguanides -------- Metformin (Glucophage, glucophage XL, Fortamet) Metformin with glyburide (glucovance) Metformin (Glucophage, glucophage XL, Fortamet) Metformin with glyburide (glucovance) action/indications -------- action/indications *inhibits production of glucose by the liver *increases body tissue's sensitivity to insulin *decreases hepatic synthesis of cholesterol *used in type 2 to control blood glucose level Metformin (Glucophage, glucophage XL, Fortamet) Metformin with glyburide (glucovance) side effects -------- side effects lactic acidosis Metformin (Glucophage, glucophage XL, Fortamet) Metformin with glyburide (glucovance) side effects -------- side effects hypoglycemia if used in combination with insulin or other antidiabetic agents drug-drug interactions Metformin (Glucophage, glucophage XL, Fortamet) Metformin with glyburide (glucovance) side effects -------- side effects contraindicated in patients with impaired renal or liver function, respiratory insufficiency, severe infection, or alcohol abuse Metformin (Glucophage, glucophage XL, Fortamet) Metformin with glyburide (glucovance)

implications -------- implications monitor for lactic acidosis and hypoglycemia Metformin (Glucophage, glucophage XL, Fortamet) Metformin with glyburide (glucovance) implications -------- implications *monitor renal function *patients taking this drug are at increased risk of acute renal failure and lactic acidosis with use of iodinated contrast material for diagnostic studies Metformin (Glucophage, glucophage XL, Fortamet) Metformin with glyburide (glucovance) implications -------- implications should be stopped 48 hours before and after use of contrast agent or until renal function is evaluated and is normal alpha-glucosidase inhibitors -------- acarbose (precose) miglitol (glyset) acarbose (precose miglitol (glyset) action/indication -------- action/indications delays absorption of complex carbs in the intestine and slows entry of glucose into systemic circulation acarbose (precose) miglitol (glyset) action/indication -------- action/indications does not increase insulin secretion used in type 2 can be used alone or in combination with sulfonylureas, metformin, or insulin to improve glucose control acarbose (precose) miglitol (glyset) side effects -------- GI side effects: abdominal discomfort or distention, flatulence, diarrhea acarbose (precose) miglitol (glyset) side effects -------- side effects hypoglycemia: risk increased if used with insulin or other antidiabetic agents acarbose (precose) miglitol (glyset) implications -------- implications must be taken with the first bite of food to be effective acarbose (precose) miglitol (glyset) implications -------- implications

monitor for GI side effects: diarrhea, abdominal distention) acarbose (precose) miglitol (glyset) implications -------- implications monitor for blood glucose levels to assess effectiveness of therapy acarbose (precose) miglitol (glyset) implications -------- implications monitor liver function studies every 3 months for 1 year, then periodically acarbose (precose) miglitol (glyset) implications -------- implications contraindicated in patients with GI or renal dysfunction or cirrhosis acarbose (precose) miglitol (glyset) implications -------- implications Alert: hypoglycemia must be treated with glucose, not sucrose non-sulfonylurea insulin secretagogues -------- repaglinide (prandin) categorized as a meglitinide nateglinide (starlix) categorized as a d-phenylalanine derivative repaglinide (prandin) categorized as a meglitinide nateglinide (starlix) categorized as a d-phenylalanine derivative action/indications -------- action/indications *stimulates pancreas to secrete insulin *used in type 2 to control blood glucose levels repaglinide (prandin) categorized as a meglitinide nateglinide (starlix) categorized as a d-phenylalanine derivative action/indications -------- action/indications *can be used alone or in combination with metformin, or thiazolidinediones to improve glucose control repaglinide (prandin) categorized as a meglitinide nateglinide (starlix) categorized as a d-phenylalanine derivative side effects -------- side effects hypoglycemia/weight gain less likely than sulfonylureas repaglinide (prandin) categorized as a meglitinide nateglinide (starlix) categorized as a d-phenylalanine derivative side effects -------- side effects drug-drug interactions with ketoconazole, fluconazole, erythromycin, rifampin, isoniazid repaglinide (prandin) categorized as a meglitinide nateglinide (starlix) categorized as a d-phenylalanine derivative implications -------- implications

*monitor blood glucose levels to assess effectiveness of therapy *has rapid action and short half-life repaglinide (prandin) categorized as a meglitinide nateglinide (starlix) categorized as a d-phenylalanine derivative implications -------- implications *should be taken only if able to eat a meal immediately *teach patients symptoms of hypoglycemia repaglinide (prandin) categorized as a meglitinide nateglinide (starlix) categorized as a d-phenylalanine derivative implications -------- implications *monitor pts with impaired liver function and renal impairment *has no effect on plasma lipids repaglinide (prandin) categorized as a meglitinide nateglinide (starlix) categorized as a d-phenylalanine derivative implications -------- implications *is taken before each meal *check for interactions with other meds thiazolidinediones (glitazones) -------- pioglitazone (actos) rosiglitazone (avandia) pioglitazone (actos) rosiglitazone (avandia) side effects -------- side effects *sensitize body tissue to insulin *stimulate insulin receptor sites to lower blood glucose and improve action of insulin pioglitazone (actos) rosiglitazone (avandia) side effects -------- side effects *may be used alone or in combination with sulfonylurea, metformin, or insulin pioglitazone (actos) rosiglitazone (avandia) side effects -------- side effects *hypoglycemia (risk increased with use of insulin or other antidiabetic drugs) *anemia pioglitazone (actos) rosiglitazone (avandia) side effects -------- side effects *weight gain *edema *decreases effectiveness of oral contraceptives pioglitazone (actos) rosiglitazone (avandia) side effects -------- side effects

*possible liver dysfunction *drug-drug interaction pioglitazone (actos) rosiglitazone (avandia) side effects -------- side effects *hyperlipidemia: has variable effect on lipids *impaired platelet function pioglitazone (actos) side effects -------- side effects may be preferred choice in patients with lipid abnormalities pioglitazone (actos) rosiglitazone (avandia) implications -------- implications *monitor blood glucose levels to assess effectiveness of therapy *monitor liver function tests *arrange dietary teaching to establish weight control program *instruct pts taking oral contraceptives about increased risk of pregnancy dipeptidyl peptidase 4 (DPP4) inhibitor -------- sitagliptin (januvia) vildagliptin (galvus) sitagliptin (januvia) vildagliptin (galvus) action/indications -------- implications increases and prolongs the action of incretin, a hormone that increases insulin release and decreases glucagon levles, with the result of improved glucose control sitagliptin (januvia) vildagliptin (galvus) side effects -------- SE *upper respiratory infection *stuffy or sunny nose and sore throat sitagliptin (januvia) vildagliptin (galvus) side effects -------- SE *headache *stomach discomfort and diarrhea *hypoglycemia, if used with sulfonylurea sitagliptin (januvia) vildagliptin (galvus) implications -------- implications *usually administered once a day *used alone or with other oral antidiabetic agents *instruct patient about signs and symptoms of hypoglycemia and other adverse effects to report *monitor renal function

Diabetic ketoacidosis (DKA) -------- A type of metabolic acidosis due to an excess of ketone bodies, most often occurring when diabetes mellitus is uncontrolled. Glucagon -------- Acts to increase blood glucose levels. Gluconeogenesis -------- The production of "new" glucose from noncarbohydrate molecules. Hyperglycemic effect -------- Causes blood glucose to rise. Hyperosmolar hyperglycemic state (HHS) -------- Acute complication seen in persons with type 2 diabetes, that is characterized by extreme hyperglycemia, hyperosmolarity with dehydration, the absence of ketoacidosis, and CNS dysfunction Hypoglycemic effect -------- Causes glucose to leave the blood and serum glucose to fall. Insulin -------- Acts to decrease blood glucose levels. Insulin analog -------- Modified human insulin with pharmacokinetic advantages, such as more rapid onset of action or prolonged duration of action. Insulin resistance -------- Occurs in type 2 diabetes mellitus; although insulin is secreted, insulin receptors in target tissues become insensitive to insulin, binding of insulin to these receptors decreases, less effect is achieved. Islets of Langerhans -------- Cell clusters in the pancreas responsible for the secretion of insulin and glucagon. Ketoacids -------- Acidic waste product of lipid metabolism that lowers the pH of the blood. Somogyi phenomenon -------- Rapid decrease in blood glucose level that stimulates the release of hormones (epinephrine, cortisol, glucagon) resulting in an elevated morning blood glucose. Type 1 Diabetes mellitus -------- Metabolic disease characterized by hyperglycemia caused by a lack of secretion of insulin by the pancreas. Type 2 Diabetes mellitus -------- Chronic metabolic disease caused by insufficient secretion of insulin by the pancreas, and a lack of sensitivity of insulin receptors. Immune-related Type 1 DM may be caused by_ -------- _genetic predisposition and exposure to a virus. Non-immune Type 1 DM is caused_ -------- _by other unknown factors. Onset of diabetes type 1: symptoms include: -------- Rapid sudden weight loss, polydipsia (thirst), polyuria, polyphagia (hunger), pt. is gaunt, fatigued, minimal or no insulin. Medication therapy for DB 1 is? -------- Insulin therapy. DB1 pt. Must have exogenous insulin for survival

How do you store insulin properly? -------- 1. Insulin may be left at room temperature. 2.Unopened vials must be stored in the refrigerator. 3.Prefilled syringes are stable for 30 days in the refrigerator. 4.Prefilled syringes should be rolled between palms to warm and mix prior to administration. Administration of Subcutaneous Insulin: -------- :If mixing, know compatibility and draw clear before cloudy Remember the two-nurse rule: -------- :two RNs must verify dose. Injection sites include: -------- :posterior upper arms, abdomen, thighs and gluteal region. Injection should be avoided_ -------- _in area which will be exercised. What size does most insulin come in? -------- Most insulin comes in 100 unit vials. Alcohol may be used _ -------- _prior to injection within a healthcare facility depending on policy For home injection_ -------- _alcohol is NOT recommended. What angle should the injection be held? -------- 45-90 degrees. What are 3 alternatives to insulin delivery? -------- Insulin pen, insulin pump, inhaled insulin. What is the most common allergic reaction to insulin? -------- A localized erythema and itching which resolves in time. What is lipodystrophy? -------- A lipodystrophy can be a lump or small dent in the skin that forms when a person keeps performing injections in the same spot. It can alter absorption. What is Somogyi Effect? -------- a rebounding high blood sugar that is a response to low blood sugar. What is dawn phenomenon? -------- a morning rise in blood sugar in response to waning insulin and a growth hormone surge What are the goals of insulin therapy? -------- Maintain glucose levels Maintain lipids and blood pressure Implement lifestyle modifications to prevent and treat obesity, Improve health through healthy food choices and exercise. Address nutritional needs while considering the client's culture, personal preferences and willingness to change.

Considerations for Type I Diabetes (3): -------- 1.The client is encourage to continue their usual intake, but should be educated about health food choices and a balances diet. 2.Insulin is dosed with eating habits and activity in mind. 3.Scheduled insulin dosing offers the least flexibility as the client must be consistent in what they eat and do. What is glycemic index? -------- a term used to describe the rise in blood sugar after a person has consumed carbohydrates. Glycemic Index > 70 =? -------- High Glycemic Index < 55 = ? -------- Low Carbohydrates - Recommended Daily Amount: -------- 45-65% of caloric intake. Fats - Recommended Daily Amount: -------- 25-30% of caloric intake. Saturated fats should be < 7%. Protein - Recommended Daily Amount: < 10% of caloric intake. -------- < 10% of caloric intake. What do the alpha cells produce in the pancreas? -------- Glucagon. What does the liver do with glucagon? -------- The liver is stimulated by glucagon to release stored glycogen as glucose into the bloodstream. What do the beta cells produce in the pancreas? -------- The beta cells produce insulin. What does insulin do? -------- Moves glucose, regulates metabolism, helps in storage of sugars, fats, and proteins; glucose to glycogen. What is normal serum glucose? -------- 70-110 mg/dL What are the 'counter-regulatory hormones? -------- Cortisol, Glucagon, Growth hormone, epinephrine. What is the definition of diabetes mellitus? -------- Diabetes is a chronic multisystem disease related to abnormal insulin production, impaired insulin utilization or both. What is the pathophysiology of Diabetes? -------- The exact cause is not known Genetic Autoimmune Viral Environmental links One or a combination

What are risk factors for type 2 diabetes? -------- Sedentary, familial tendency, avg. age 50 yrs., Hx of high BP, obesity, infections, FBS > 126 mg/dL, polyuria, poydipsia What are the common diagnostic studies? -------- Fasting blood glucose, random plasma glucose, glycosylated hemoglobin (A1c), Oral glucose tolerance test The average adult produces how much insulin per day? -------- 40-50 U of insulin per day How many US citizens have DB? -------- 21 million How many US citizens are pre-diabetic? -------- 40 million Regarding blindness, diabetes is__ -------- __the leading cause. People with diabetes run twice the risk for getting__ -------- __heart disease. What is the definition of hypoglycemia? -------- blood sugar below 70 mg/dL Signs and symptoms of hypoglycemia? -------- Confusion, diaphoresis, tremors, hunger, weakness, visual changes Hospital management of acute hypoglycemia: -------- 1.In hospital setting if it is below 40, repeat accu check and then obtain stat lab. 2. If able to swallow and intact gag reflex treat immediately with 15 grams of carbohydrates orally 3. If decreased LOC and client unable to swallow give 1 ampule (50mL) dextrose 50% (D50) or 1 mg glucagon IM. 4. Repeat glucose level in 15 min if level is less than 70 mg/dl repeat If > 70 mg/dl give at least an additional 15 grams of carbohydrate plus fat or protein Hypoglycemic unawareness - -------- When a client does not feel or experience the warning signs of hypoglycemia. Diabetic Ketoacidosis (DKA) -------- extreme insulin deficiency leads to hyperglycemia ketosis -------- condition resulting from uncontrolled diabetes mellitus, in which the body has an abnormal concentration of ketone bodies resulting from excessive fat metabolism acidosis -------- condition brought about by an abnormal accumulation of acid products of metabolism seen in uncontrolled diabetes mellitus Causes of DKA: -------- insufficient insulin, illness, infection, poor management of DM and undiagnosed DM Type I. S & S of DKA -------- Dehydration, Polyuria, Abdominal pain, vomiting, anorexia, weakness, fatigue, neuro changes, Kussmaul's resps., fruity breath, hyperglycemia, acidosis, ketones, glucoseuria, electrolyte imbalances. Diabetes mellitus is _ -------- _a chronic multisystem disorder of glucose metabolism related to absent or insufficient insulin supplies, impaired utilization of insulin, or both. Current theories link the causes of diabetes to _ -------- _genetic, autoimmune, and environmental factors.

Type 1 diabetes mellitus most often occurs _ -------- _in people who are under 30 years of age, with a peak onset between ages 11 and 13, but can occur at any age. Type 1 diabetes is the _ -------- _end result of a long-standing process where the body's own T cells attack and destroy pancreatic beta cells, which are the source of the body's insulin. Because the onset of type 1 diabetes is rapid_ -------- _, the initial manifestations are usually acute. The classic symptomspolyuria, polydipsia, and polyphagia_ -------- _are caused by hyperglycemia and the accompanying spillover of excess glucose in the urine. The individual with type 1 diabetes requires _ -------- _a supply of insulin from an outside source, such as an injection, in order to sustain life. Without insulin, the patient will develop_ -------- _diabetic ketoacidosis (DKA), a lifethreatening condition resulting in metabolic acidosis. Prediabetes is a condition in which _ -------- _blood glucose levels are higher than normal but not high enough for a diagnosis of diabetes. Those with prediabetes will usually develop _ -------- _type 2 diabetes within 10 years if no preventive measures are taken. Long-term damage to the body, especially the heart and blood vessels_ -------- _may already be occurring in patients with prediabetes. Type 2 diabetes mellitus accounts for _ -------- _over 90% of patients with diabetes. In type 2 diabetes, the pancreas _ -------- _usually continues to produce some endogenous (self-made) insulin. However, the insulin that is produced is either insufficient for the needs of the body and/or is poorly used by the tissues. The most important risk factor for developing type 2 diabetes is believed to be _ -------_obesity, specifically abdominal and visceral adiposity. The manifestations of type 2 diabetes are more nonspecific, and include _ -------_fatigue, recurrent infections, recurrent vaginal yeast or monilia infections, prolonged wound healing, and visual changes. Gestational diabetes develops _ -------- _during pregnancy and is detected at 24 to 28 weeks of gestation, usually following an oral glucose tolerance test. Although most women with gestational diabetes will have normal glucose levels within 6 weeks postpartum_ -------- _their risk for developing type 2 diabetes in 5 to 10 years is increased.

A diagnosis of diabetes is based on _ -------- _one of three methods: fasting plasma glucose level, random plasma glucose measurement, or 2-hour oral glucose tolerance test. The glycosated hemoglobin test (HbA1c) is useful _ -------- _in evaluating long-term glycemic levels. The American Diabetes Association (ADA) recommends keeping the A1C level below 7%. The goals of diabetes management are to_ -------- _reduce symptoms, promote wellbeing, prevent acute complications of hyperglycemia, and prevent or delay the onset and progression of long-term complications. These goals are most likely to be met when the patient is able to maintain blood glucose levels as near to normal as possible. Exogenous (injected) insulin is needed when _ -------- _a patient has inadequate insulin to meet specific metabolic needs. Insulin is divided into two main categories: _ -------- _short-acting (bolus) and longacting (basal) insulin. Basal insulin is used to _ -------- _maintain a background level of insulin throughout the day; bolus insulin is used at mealtime to combat postprandial hyperglycemia and at bedtime. A variety of insulin regimens are recommended for patients _ -------- _depending on the needs of the patient and their preference. Routine administration of insulin is most commonly done by means of _ -------_subcutaneous injection, intravenous administration of regular insulin can be done when immediate onset of action is desired. The technique for insulin injections _ -------- _should be taught to new insulin users and reviewed periodically with long-term users. The speed with which peak serum concentrations are reached _ -------- _varies with the anatomic site for injection. The fastest absorption is from the abdomen. Continuous subcutaneous insulin infusion _ -------- _can be administered using an insulin pump, a small battery-operated device that resembles a standard paging device in size and appearance. An insulin pump is programmed to deliver_ -------- _a continuous infusion of short-acting insulin 24 hours a day with boluses at mealtime. Hypoglycemia, allergic reactions, lipodystrophy, and Somogyi effect are _ -------_problems associated with insulin therapy. o Lipodystrophy (atrophy of subcutaneous tissue) may occur _ -------- _if the same injection sites are used frequently but its incidence has decreased with the use of human insulin.

o The Somogyi effect is _ -------- _a rebound effect in which an overdose of insulin induces hypoglycemia. o Usually occurring during the hours of sleep_ -------- _the Somogyi effect produces a decline in blood glucose level in response to too much insulin. o The dawn phenomenon is characterized by _ -------- _hyperglycemia that is present on awakening in the morning resulting from the release of counterregulatory hormones in the predawn hours. Oral agents (OAs) are not insulin; they work to _ -------- _to improve the mechanisms by which insulin and glucose are produced and used by the body. OAs work on the three defects of type 2 diabetes_ -------- _, including insulin resistance, decreased insulin production, and increased hepatic glucose production. Sulfonylureas are frequently the drugs of choice in treating _ -------- _type 2 diabetes because of the decreased chance of prolonged hypoglycemia. The primary action of the sulfonylureas is to_ -------- _increase insulin production from the pancreas. Meglitinides increase_ -------- _insulin production from the pancreas. Because they are more rapidly absorbed and eliminated_ -------- _, meglinitiedes offer a reduced potential for hypoglycemia. Metformin (Glucophage) is a_ -------- _biguanide glucose-lowering agent. The primary action of metformin is to _ -------- _reduce glucose production by the liver. -Glucosidase inhibitors, also known as "starch blockers," _ -------- _work by slowing down the absorption of carbohydrate in the small intestine. Sometimes referred to as "insulin sensitizers," _ -------- _thiazolidinediones are most effective for people who have insulin resistance. Thiazolidinediones improve_ -------- _insulin sensitivity, transport, and utilization at target tissues. Pramlintide (Symlin) is_ -------- _a synthetic analog of human amylin, a hormone secreted by the cells of the pancreas. When taken concurrently with insulin_ -------- _ Pramlintide (Symlin) provides for better glucose control. Exenatide (Byetta) is _ -------- _a synthetic peptide that stimulates the release of insulin from the pancreatic cells. Exenatide is administered_ -------- _ using a subcutaneous injection.

The overall goal of nutritional therapy is_ -------- _to assist people with diabetes in making healthy nutritional choices, eating a varied diet, and maintaining exercise habits that will lead to improved metabolic control. For those with type 1 diabetes_ -------- _day-to-day consistency in timing and amount of food eaten is important for those individuals using conventional, fixed insulin regimens. Patients using rapid-acting insulin _ -------- _can make adjustments in dosage before the meal based on the current blood glucose level and the carbohydrate content of the meal. The emphasis for nutritional therapy in type 2 diabetes should be placed on_ -------_achieving glucose, lipid, and blood pressure goals. The nutritional energy intake should be constantly balanced _ -------- _with the energy output of the individual, taking into account exercise and metabolic body work. In a general diabetic meal plan_ -------- _carbohydrates and monounsaturated fat should provide 45% to 65% of the total energy intake each day. Fats should compose no more than 25% to 30% of the meal plan's total calories_ -------_with less than 7% of calories from saturated fats and protein contributing less than 10% of the total energy consumed. Alcohol is _ -------- _high in calories, has no nutritive value, and promotes hypertriglyceridemia. Patients should be cautioned to _ -------- _honestly discuss the use of alcohol with their health care providers because its use can make blood glucose more difficult to control. Regular, consistent exercise _ -------- _is considered an essential part of diabetes and prediabetes management. Exercise increases insulin receptor sites in the tissue and can have a direct effect on lowering the blood glucose levels. Self-monitoring of blood glucose (SMBG) _ -------- _is a cornerstone of diabetes management. By providing a current blood glucose reading, SMBG enables the patient to make self-management decisions regarding diet, exercise, and medication. The frequency of monitoring depends on several factors_ -------- _including the patient's glycemic goals, the type of diabetes that the patient has, the patient's ability to perform the test independently, and the patient's willingness to test. Pancreas transplantation can be used _ -------- _as a treatment option for patients with type 1 diabetes mellitus. Most commonly, pancreas transplantation is done for patients who have_ -------- _ endstage renal disease and who have had or plan to have a kidney transplant. Kidney and pancreas transplants are often performed together_ -------- _or a pancreas may be transplanted following a kidney transplant. Pancreas transplants alone are rare.

Nursing responsibilities for the patient receiving insulin include _ -------- _proper administration, assessment of the patient's response to insulin therapy, and education of the patient regarding administration of, adjustment to, and side effects of insulin. The goals of diabetes self-management education are to _ -------- _enable the patient to become the most active participant in his or her care, while matching the level of selfmanagement to the ability of the individual patient. Diabetic ketoacidosis (DKA), also referred to as _ -------- _"diabetic acidosis" and "diabetic coma," is a life-threatening condition caused by a profound deficiency of insulin . Diabetic ketoacidosis (DKA)_ -------- _is characterized by hyperglycemia, ketosis, acidosis, and dehydration. It is most likely to occur in people with type 1 diabetes. Hyperosmolar hyperglycemic syndrome (HHS) _ -------- _is a life-threatening syndrome that can occur in the patient with diabetes who is able to produce enough insulin to prevent DKA but not enough to prevent severe hyperglycemia, osmotic diuresis, and extracellular fluid depletion. Hypoglycemia, or low blood glucose, occurs when _ -------- _there is too much insulin in proportion to available glucose in the blood. o Causes of hypoglycemia are often related to _ -------- _a mismatch in the timing of food intake and the peak action of insulin or oral hypoglycemic agents that increase endogenous insulin secretion. o A critical role of the nurse is the prompt recognition of hypoglycemia and _ -------_initiating the appropriate treatment dependent on the patient's status. Chronic complications of diabetes are _ -------- _primarily those of end-organ disease from damage to blood vessels as a result of chronic hyperglycemia. The chronic blood vessel dysfunctions of diabetes are divided into two categories:_ -------- _macrovascular complications and microvascular complications. o Macrovascular complications are_ -------- _diseases of the large and medium-sized blood vessels that occur with greater frequency and with an earlier onset in people with diabetes. o Microvascular complications result from _ -------- _thickening of the vessel membranes in the capillaries and arterioles in response to conditions of chronic hyperglycemia. Diabetic retinopathy refers to _ -------- _the process of microvascular damage to the retina as a result of chronic hyperglycemia in patients with diabetes. There are two types of diabetic retinopathy:_ -------- _ proliferative and nonproliferative. Because the earliest and most treatable stages produce no vision changes, persons with diabetes should have an annual dilated eye examination.

Diabetic nephropathy is _ -------- _a microvascular complication associated with damage to the small blood vessels that supply the glomeruli of the kidney. Patients should be screened for diabetic nephropathy annually with _ -------- _a measurement of albumin-creatinine ratio from a urine specimen. Diabetic neuropathy is nerve damage that occurs _ -------- _because of the metabolic derangements associated with diabetes mellitus. The two major categories of diabetic neuropathy are _ -------- _sensory neuropathy, which affects the peripheral nervous system, and autonomic neuropathy. The most common form of sensory neuropathy is _ -------- _distal symmetric neuropathy, which affects the hands and/or feet bilaterally. This is sometimes referred to as "stocking-glove neuropathy." Autonomic neuropathy can affect _ -------- _nearly all body systems and lead to hypoglycemic unawareness, bowel incontinence and diarrhea, and urinary retention. Foot complications are _ -------- _the most common cause of hospitalization in the person with diabetes. Sensory neuropathy is a major risk factor for lower extremity amputation in the person with diabetes. Proper care of a diabetic foot ulcer _ -------- _is critical to prevent infections. Loss of protective sensation _ -------- _often prevents the patient from becoming aware that a foot injury has occurred. Common skin complications in diabetes include _ -------- _Acanthosis nigricans, diabetic dermatopathy, Necrobiosis lipoidica diabeticorum, and Granuloma annulare. Because the thin skin is prone to injury_ -------- _special care must be taken to protect affected areas from injury and ulceration. A patient with diabetes is more susceptible to infections because of _ -------- _a defect in the mobilization of inflammatory cells and an impairment of phagocytosis by neutrophils and monocytes. The treatment of infections must be prompt and vigorous _ -------- _since antibiotic therapy has prevented infection from being a major cause of death in diabetic patients. Patients with diabetes have high rates_ -------- _of psychiatric disorders, particularly depression. Patients need to be assessed for the signs and symptoms of depression at each visit. Women with type 1 diabetes have an increased risk _ -------- _of developing an eating disorder in comparison to women without diabetes. Open and collaborative communication is critical for identifying these behaviors early. NPH(Humulin N, Novolin N, ReliOn N) -------- intermediate

Lantus (glargine) -------- long-acting detemir(Levemir) -------- long-acting lispro, aspart, flulisine -------- rapd-acting Regular(Humulin R, Novolin R, ReliOn R) -------- short-acting high BG symptoms (hyperglycemia) -------- thirst, hunger, frequent urination, fatigue, nausea, blurred vision, HA, nervousness, confusion low BG symptoms (hypoglycemia) -------- shakiness, sweaty, hunger, anxiety, nervousness, confusion, acting angry or irritable, slurred speech, HA Causes of hyperglycemia -------- too much food, too little exercise, too little medicine, stress, illness, injury, short time between meals and snacks Causes of hypoglycemia -------- too little food, too much medicine, more activity than usual, too long between meals or snacks, alcohol mild hypoglycemia -------- 60-70 mg/dL moderate hypoglycemia -------- 41-59 mg/dL severe hypoglycemia -------- <40 mg/dL relative hypoglycemia -------- >70 mg/dL Rapid-acting -------- Onset: 15 minutes Peak: 60-90 minutes Duration: 3-4 hours Short-acting -------- Onset: 30-60 minutes Peak: 2-3 hours Duration: 3-6 hours Intermediate-acting -------- Onset:2-4 hours Peak: 4-10 hours Duration: 10-16 hours Long-acting -------- Onset: 1-2 hours Peak: no pronounced peak Duration: 24+ hours NPH/regular 70/30 -------- Humulin 70/30, Novolin 70/30, ReliOn 70/30 NPH/regular 50/50 -------- Humulin 50/50, lispro protamine/lispro 75/25 -------- Humalog mix 75/25 aspart protamine/aspart 70/30 -------- Novolog Mix 70/30

sulfonylureas -------- stimulate release of insulin from pancreatic islets; decrease glucogenolysis and gluconeogenesis; enhance cellular sensitivity to insulin. increases insulin secretions meglitinides -------- stimulate a rapid and short-lived release of insulin from the pancreas biguanide -------- decreases rate of hepatic glucose production; augments glucose uptake by tissues, esp. muscles (lower risk of hypoglycemia) a-Glucosidase inhibitors -------- delay absorption of glucose from GI tract thiazolidinedioness -------- glucose uptake in muscle; decreases endogenous glucose production (timing for meals not as important) Dipeptidyl Peptidase-4 (DDP-4) inhibitors -------- enhances the incretin system, stimulates release of insulin from pancreatic B bells, and decreases hepatic glucose production Glucovance -------- combination of metformin and glyburide Avandamet -------- combination of rosiglitazone and metformin Metaglip -------- combination of metformin and glipizide Duetact -------- combination of pioglitazone and glimepiride incretin mimetic -------- stimulates release of insulin; decreases glucagon secretion; increases satiety, decreases gastric emptying amylin analog` -------- decreases gastric emptying; decreases glucagon secretion; decreases endogenous glucose output from liver; increases satiety which oral med classifications are given before meals? -------- sulfonylureas, meglitinides which oral med classification is faster than sulfonylureas? -------- meglitinides which classification do you hold if they don't eat? -------- meglitinides which classification helps the "leaky faucet" -------- giguanide which type do you take with first bite of food (starch blocker)? -------- a-Glucosidase inhibitors which type is for insulin resistance, makes insulin work better? -------- thiazolidinediones type 1 drug therapy -------- 1 from basal, 1 from bolus type 2 drug therapy -------- 1 from basal OR 1 from bolus type 1 (oral drug therapy) -------- no oral insulin

type 2 -------- shot of long acting and oral pill to cover meals when mixing insulin in syringe remember -------- clear to cloudy do not mix -------- long-acting insulin which drug therapy is dose and eat or eat and dose -------- rapid-acting which drug therapy mimics postprandial rise and fall of insulin -------- rapid-acting intermediate-acting is usu. given _____ time(s) a day -------- 2 diabetes mellitus -------- a chronic multisystem disease related to abnormal insulin production, impaired insulin utilization or both What makes diabetes such a devastating disease? -------- the long term complications associated with it Before insulin was discovered, how was diabetes treated? -------- a diet of salad and eggs insulin -------- a hormone produced by the beta cells in the islets of Langerhans of the pancreas How does insulin work (under normal circumstances) to reduce blood glucose levels? -------- insulin is continuously released into blood stream in small pulsatile increments (basal rate) and increased release (bolus rate) when food is ingested, and works to lower the blood glucose level by working as a "key" to transport sugar from blood stream across the cell membrane to the cytoplasm of the cell, where it can be utilized for energy; stimulates glucose storage in the liver and muscle in the form of glycogen Under normal circumstances, what is the range that blood glucose levels are maintained at all times? -------- 70-120 mg/dl What is the average amount of insulin secreted daily by an adult under normal circumstances? -------- 40-50 units counterregulatory hormones -------- hormones that work to oppose the effects of insulin; include glucagon, epinephrine, growth hormone and cortisol How do counterregulatory hormones prevent a person from becoming hypoglycemic? -------- they increase blood glucose levels by stimulating glucose production and output by the liver; they also decrease the movement of glucose into the cells What are the four theories linked to the cause of diabetes mellitus? -------- genetic, autoimmune, viral, and environmental Where is glucagon produced? -------- by the alpha cells in the pancreas type I diabetes -------- the end result of a long standing process in which the body's own T cells attack and destroy pancreatic beta cells, which are the source of the body's insulin (autoimmune disease); can also be the result of trauma/disease affecting the pancreas,

or an infection that has caused damage to the pancreas; manifests itself when a person can no longer produce insulin; onset of symptoms is usually rapid, but disease process has been ongoing long before symptoms are evident What are some of the symptoms manifested on onset of diabetes type I? -------- sudden weight loss, polydipsia, polyuria, polyphagia, weakness/fatigue diabetic ketoacidosis -------- a life threatening condition resulting in metabolic acidosis; prevented/treated with administration of an outside source of insulin glycogenolysis -------- the breaking down of glucose type II diabetes mellitus -------- most prevealent type of diabetes; occurs when the body cannot use the insulin it makes (receptors have changed so the key/insulin does not work-insulin resistance); can also occur if the insulin produced is not sufficient to decrease the amount of glucose in the blood; can also occure when the liver does not produce enough glucose; gradual onset usually associated with poor eating habits and sedentary lifestyle Why is type II diabetes now being seen more often in children and adolescents? -------this is due to the epidemic of early childhood obesity and the sedentary lifestyles of children What are some factors that would increase an individuals risk of getting type II diabetes? -------- genetics (family history); above age 35; obesity (especially abdominal); insulin resistance; high levels of insulin in blood, increased LDL; increased triglycerides; decreased HDL Which ethnic groups have a higher risk of type II diabetes? -------- Native Americans, Alaska Natives, African americans, Hispanic Americans, and Asian Americans prediabetes -------- diagnosed when fasting glucose levels are higher than normal (100 mg/dl) but lower than 126 mg/dl; people with prediabetes usually develop type II diabetes within 10 years; treat with weight loss and exercise to prevent onset of type II diabetes gestational diabetes -------- develops during pregnancy; will have normal glucose levels within 6 weeks post partum; risk for developing type II diabetes in 5 to 10 years is increased; treated with nutritional therapy and then insulin therapy if that is not sufficient secondary diabetes -------- occurs in some people because of another medical condition or due tot he treatment of a medical condition that causes abnormal blood glucose levels; treated by treating the underlying cause/condition What three diagnostic tests are used to diagnose diabetes? -------- fasting plasma glucose level testing, random/casual plasma glucose measurement, and two hour OGTT level testing fasting plasma glucose level testing -------- done with no caloric intake for at least eight hours prior; confirmed by repeat testing on another day; preferred method of diagnosis;

if glucose level is equal to or greater than 126 on two separate occasions, person is diagnosed with diabetes random/casual plasma glucose level testing -------- done anytime of day without regard to the time of the last meal; if glucose level is above 200mg/dl AND the person exhibits manifestations of diabetes ( polyuria, polydipsia, and unexplained weight loss) they are diagnosed with diabetes two hour OGTT level -------- accuracy depends on adequate patient preparation and attention to factors that could influence outcome like severe restriction of carbs, acute illness, medications, and restricted activity; if patient has blood glucose levels of above 200 mg/dl they are diagnosed with diabetes How often should a person over the age of 45 be screened for diabetes? -------- every 3 years How often should a person with a BMI of greater than 25 be screened for diabetes? -------every 3 years What are some other circumstances that would require a person under the age of 45 be screened for diabetes every 3 years? -------- they have a first degree relative with diabetes, they life an inactive lifestyle (bedrest), had a baby weighing over 9 lbs, high cholesterol levels hemoglobin A1c -------- a test done that shows the amount of glucose attached to hemoblogin molecules over their lifespan (3 months) What is the recommended hemoglobin A1c percentage for a patient with diabetes? -------- less than 7 % (some say 6.5%) What is teh recommended hemoglobin A1c percentage for a patient without diabetes? -------- between 4-6% Why is the hemoglobin A1c considered the most accurate measurement of glucose control? -------- Because it loos at the bigger picture (across a 3 month span) What should be included in the eduation of a diabetic patient? -------- medication, food intake (also teach person preparing patient's meals), exercise, blood glucose testing rapid-acting insulin -------- used to control postmeal blood glucose levels; onset is 15 minutes; peak is 60-90 minutes; duration is 3-4 hours; include lispro, aspart, and glulisine short-acting insulin -------- used to control postmeal blood glucose levels; onset is 1/2-1 hr; peak is 2-3 hrs; duration is 3-6 hrs; regular insulin How long before a meal should rapid-acting insulin be injected? -------- 0-15 min How long before a meal should short-acting insulin be injected? -------- 30-45 min glyconeogenesis -------- making more glucose

intermediate-acting insulin -------- used in combination therapy to control blood glucose levels between meal times and overnight; can be mixed with short or rapid acting insulin to avoid more injections; more appealing to patient than long acting (which can not be mixed with short/rapid acting in the same injection) but may not achieve sufficient blood glucose control; onset time is 2-4 hrs; peak is 4-10 hrs; duration 10-16 hrs; NPH insulin very long-acting insulin -------- used in addition to bolus insulin, but can not be mixed in same syringe; controls glucose levels between mealtimes and overnight; onset is 1-2 hrs; no pronounced peak; duration is 24+ hrs; glargine and detemir; can not prefill syringes with glargine, must be filled right before administration combination insulins -------- premixed by the manufacture; 70/30 NPH/regular (composed of 70% NPH and 30% regular); 50/50 nph/regular; 75/25 lispro protamine/lispro; 70/30 aspart protamine/aspart What is the goal of insulin therapy? -------- to mimic normal pancreatic insulin production What is the key to successful blood glucose control? -------- self blood glucose monitoring What should a nurse ALWAYS do before administering insulin? -------- check the vial and dosage with another nurse while needle is still in syringe long-acting insulin -------- used to control glucose levels between meals and overnight; onset is 30min-3 hrs; peak is 10-20 hrs; duration is up to 36 hrs; ultralente insulin How does sliding scale administration work? -------- a predetermined amount of insulin to be administered is based on the blood glucose level of the patient at that time What should be included in the education of the patient about insulin administration? -------- wash hands and inspect vial; gently roll the vial to mix suspensions; rotate sites within one anatomical area (to keep absorption rate more consistent between administrations); store prefilled syringes for up to 30 days with needles pointing up; can store unopened vials at room temperature for up to 4 weeks Can insulin be taken orally? -------- no because it is a protein and the digestive enzymes would denature it intensive insulin therapy -------- insulin therapy that requires multiple daily injections of insulin and frequent self monitoring of blood glucose levels (4-6 times per day) Which type of insulin can be given IV? -------- regular insulin when immediate onset is desired If a patient is given 70/30 NPH/regular insulin at 7am, when are they most likely to have a hypoglycemic episode? -------- between 4 and 6 pm What are some problems sometimes seen with insulin therapy? -------- hypoglycemia; allergic reaction to insulin; lipodystrophy; somogyi effect; dawn phenomenon

allergic reactions to insulin -------- local inflammatory reaction to insulin injection site; itching, erythema, and burning; may be self limiting within 1-3 months or may be treated with a low dose of antihistamine; true allergic reaction is rare and more often associated with preservatives used and in the latex stoppers on the vials rather than in the insulin itself lipodystrophy -------- atrophy of subcutaneous tissue and may occur if hte same injection site is used too frequently; prevented by rotation of injection sites hypertrophy -------- hardening of the subcutaneous tissue associated with lipodystrophy, regresses if patient does not use site for at least six months; can cause decreased absorption of insulin administered somogyi effect -------- nightime hypoglycemia caused by insulin overdose; counterregulatory hormones cause high blood sugar in the am; can result in patient or provider increasing insulin dosage in the morning, causing more hypoglycemic effect; treated by decreasing insulin dosage given at night dawn phenomenon -------- counterregulatory hormones secreted during sleep cause high am blood sugar; glowth hormones and cortisol play a role in this phenomenon; treated by patient eating a snack before bedtime to keep blood sugar from dropping overnight oral agents -------- not insulin; work to improve the mechanisms by which insulin and glucose are produced and used in the body; used to treat type II diabetes sulfonylureas -------- drug of choice for treating type II because of the decreased chance of prolonged hypoglycemia; increases insulin production from the pancreas; 10% of patiets experience decreased effectiveness meglitinides -------- increases insulin production from the pancreas; more rapidly absorbed and eliminated than sulfonylureas; should be taken anytime from 30 minutes before meal; should NOT be taken is meal is skipped biguanides -------- help insulin's action at the receptor site on the cell membrane and reduces glucose production by the liver; does not promote weight gain alpha glucosidase inhibitors -------- aka "starch blockers"; work by slowing down the absorption of carbohydrates in the small intestines; taken with the first bite of each meal thiazolidinediones -------- most effective in those with insulin resistenance; improves insulin sensitivity, transport, and utilization at target tissues; can cause edema-not to be used in patients with CHF dipeptidyl peptidase-4 inhibitors (DPP4's) -------- work by increasing and prolonging incretin levels; responds to elevated glucose levels; lowers glucose production by the liver; lower potential for hypoglycemia than some agents incretin -------- a hormone produced by the intestine which increases insulin synthesis and release from the pancreas What class of drugs are glipizide and glimepiride in? -------- sulfonylureas

What class of drugs are repaglinic and nateglinide in? -------- meglitinides What class of drugs is metformin in? -------- biguanides What class of drugs is acarbose in? -------- alpha glucosidase inhibitors What class of drugs are pioglitazone and rosiglitazone in? -------- thiazolidinediones What are the three drugs combined with metformin in combination therapy tablets? -------- glyburide, rosiglitazone and glipizide What must you remember about metformin? -------- can not be given for 48 hours after IV contrast has been given or 24 hours prior What is the most important thing to remember about nutritional therapy for a type I diabetic patient -------- CONSISTENCY in timing and amount of food because of the insulin therapy What is important to emphasize when education a type II diabetic patient about nutritional therapy? -------- glucose, lipid and blood pressure goals as well as carlorie/fat reduction What is the plate method? -------- 1/2 non starchy veggie; 1/4 starchy veggie; 1-4 protein; glass of nonfat milk; 1 serving of fresh fruit glycemic index -------- term used to describe the rise in blood glucose after a person has consumed a carbohydrate What is involved in patient teaching surrounding exercise? -------- should be consistent; should check blood glucose before during and after (especially if starting a new program); increase insulin receptor sights and have a direct effect on lowering blood glucose and weight loss; best scheduled 1 hour after meal; keep snacks available every 30 minutes during exercise; delay or modify exercise if BG is above 250 What is involved in patient teaching surrounding self blood glucose monitoring? -------keep a dated written record; adjust treatment regimen to obtain optimal control; check 2-3 times/day; before meals bedtime and anytime if hypoglycemia is suspected glucose urine testing -------- not accurate due to renal threshold ketones in urine -------- signals risk for ketoacidosis; by product of fat breakdown in the absence of insulin; test for ketones when blood glucose level is over 240 for two tests, illness, diabetic or pregnant What should I do if I'm sick and diabetic? -------- continue regular meal plan; increase noncaloric fluids and decaffeinated beverages; continue oral agents and insulin as prescribed; check glucose every 4 hours diabetic ketoacidosis diagnostics -------- blood pH less than 7.3; decreased bicarbonate level; ketones present in urine

What symptoms are related to the fluid shift resulting from osomotic effects of high blood glucose? -------- polydipsia and polyuria What are the signs and symptoms of DKA? -------- polyuria, polydipsia, polyphagia, lethargy, coma, dehydration, acidosis, acetone breath, kussmaul's respirations How is DKA treated and managed? -------- ABC's first; rehydration with saline until output is 30-60ml per hour; restore electrolytes; give insulin (after saline); give sodium bicarbonate if blood pH is less than 7.3 hyperosmolar hyperglycemic syndrome -------- seen in type II diabetes; enough insulin produced to prevent DKA, but sever hyperglycemia, osmotic diuresis and extracellular fluid depletion is needed; treated as a medical emergency with high mortality rate; treated similar to DKA What are the signs and symptoms of hyperosmolar hyperglycemic syndrome? -------- very high blood glucose levels; marked increase in serum osmolality; impaired thirst or inability to replace fluids; neurological changes; hypotension; dehydration; tachycardia; ketone bodies absent or minimal hypoglycemia -------- low blood glucose; occurs when there is not enough food intake, too much insulin, or too much exercise; characterized by a blood glucose level of less than 70 What are the signs and symptoms of hypoglycemia? -------- confusion, irritability, sweating, tremors, weakness, visual disturbances, unconsciousness, seizures, coma and death How is hypoglycemia treated? -------- check blood glucose; immediately ingest 15-20 gm of a simple carb; 4-6 oz of oj, soft drink, 8oz of milk; avoid fat containing sweets (take longer to absorb); recheck blood glucose in 45 min.; may be treated with 20-50 ml of 50% dextrose IV push macrovascular complications -------- diseases of the large and medium size blood vessels that occur with greater frequency and with an earlier onset in people with diabetes; atherosclerosis; increased risk of heart attack, stroke and amputations How can macrovascular complications be prevented? -------- by not smoking, controlling hypertension, decreasing fat intake, losing weight and being active microvascular complications -------- result from the thickening of the vessel membranes in the capillaries and arterioles in response to conditions of chronic hyperglycemia; specific to diabetes diabetic retinopathy -------- the process of microvascular damage to the retina as a result of chronic hyperglycemia in patients with diabetes non proliferative retinopathy -------- most common form of retinopathy; partial occlusion of the small blood vessels in the retina causes the development of microaneurysms in teh capillary walls

proliferative retinopathy -------- the most severe form; involves the retina and vitreous; when retinal capillaries become occluded, the body compensates by forming new blood vessels to supply the retina with blood; these new blood vessels are fragile and hemorrhage easily diabetic nephropathy -------- a microvascular complication associated with damage to the small blood vessels that supply the glomeruli of the kidney; kidney's filtration mechanism is stressed, holes and fibrous tissue develop; protein leaks out into urine diabetic neuropathy -------- nerve damage that occurs because of the metabolic derangements associated with diabetes mellitus sensory neuropathy -------- loss of sensation; pain; abnormal sensation; complete or partial loss of sensitivity to touch and temperature is common; can cause atrophy of the small muscles of the hands and feet What is the treatment for sensory neuropathy? -------- control of blood glucose, pain medications, capsaicin, tricyclic antidepressants autonomic neuropathy -------- affects nearly all body systems and lead to hypoglycemic unawareness, bowel incontinence, and diarrhea/urinary retention gastroparesis -------- can cause nausea, vomiting, reflux and delayed absorption of food, hypoglycemia (can't tell they are hungry/ don't eat) What is the most common cause of hospitalization of people with diabetes? -------- foot complications What are some risk factors for developing foot complications? -------- sensory neuropathy; peripheral arterial disease, clotting abnormalities, impaire immune function, autonomic neuropathy, smoking, improper footwear, going barefoot How are foot complications prevented? -------- prescription footwear; diligent skin and nail care; inspect foot thoroughly each day; treat small problems promptly Why are diabetic people more susceptible to infection? -------- decreased circulation leads to a defect in the mobilization of inflammatory cells; impaired phagocytosis; neuropathy delays detection Why does prevalence of diabetes increase with age? -------- reduction of beta cell function; decreased insulin sensitivity; altered metabolism of carbohydrates; likely to be recieving medications that can impair action of insulin alpha cells -------- glucagon beta cells -------- insulin delta cells -------- somatastatin, inhibits both insulin and glucagon

glucagon -------- used for energy production, increases blood sugar insulin -------- key to the cell, decreases blood sugar IDDM or type 1 diabetes -------- insulin dependant, pancreas does not secrete insulin what causes IDDM? -------- autoimmune disorder NIDDM or type 2 diabetes -------- controlled mostly by diet and exercise, oral meds, body produces insulin but doesn't work right impaired glucose intolerance -------- BS above normal without meeting other DM requirement, BS is usually around 140-200 IFG(impaired fasting glucose) -------- results when a fasting BS is 110-126 Syndrome X -------- metabolic disorder, insulin resistance, glucose intolerance, low HDL's, high triglycerides, HTN, abd obesity, will lead to DM, need lifestyle change, diet change, exercise other causes of DM -------- chronic pancreatitis, prolonged use of steroids, diuretics, and thyroid meds MODY(maturity onset of diabetes of the young) -------- inherited insulin impairment that occurs before age 25 polydipsia -------- excessive thirst polyuria -------- exessive urination polyphagia -------- excessive hunger glycosuria -------- glucose in the urine S/S of diabetes -------- polydypsia, polyuria, polyphagia, glycosuria, fatigue, blurred vision, abd pain, headaches random blood glucose -------- no prep, greater than 200 suggests DM fasting blood glucose -------- NPO for 8 hrs postpradial glucose -------- taken 2-4 hrs after meals, DM suggest if BG is greater than 140 oral glucose tolerance test -------- high sugar drink, tests urine and blood every hour for 3 hrs glycosylated hemoglobin Hgb A1C -------- lab draw to check BS for the last 90-120 days, above 8 percent indicates DM or poorly controlled DM treatment for DM -------- pancreas transplant

retinopathy -------- blood vessel damage of the eyes, leads to blindness, should avoid strenuous exercise, side effect of DM neuropathy -------- nerves become affected caused by high blood sugar, can't feel legs or arms well just tingling, don't know when they get hurt how does sickness affect BS? -------- it increases it onset -------- the start or beginning of insulin working peak -------- when insulin is at it's highest, working the hardest duration -------- length of time insulin is effective 1 unit per hour -------- amount of insulin a normal pancreas secretes lantus, levmir -------- can't be mixed with any other insulin, given once in 24 hrs, doesn't have a peak time humalog, novalog -------- rapid acting insulin, onset 15-30 min, peak 30-90 min, duration less than 5 hours, clear regular insulin, humulin R or novalin R -------- short acting insulin, onset 30-60 min, peak 2-5 hrs, duration 5-8 hrs, clear NPH, humulin N, novalin N -------- intermediate acting insulin, onset 1 1/2-2 hrs, peak 412 hrs, duration 24 hrs, cloudy Lantus, levmir -------- long acting insulin, onset 2-4 hrs, no peak, duration 24 hrs, clear how should you mix insulin -------- clear to cloudy, air in cloudy first, clear what insulin is the only one that can be given IV? -------- regular, humulin R or novalin R insulin sites -------- back of arm, abd, fatty part of leg, love handles, rotate sites for effectiveness insulin pump -------- closest thing to normal pancreas function, sub q needle stays in abd, delivers insulin at a basil rate plus allows you to bolus sliding scale -------- insulin dose is based on blood sugar results, based on a range, usually uses shorter and faster acting insulins like regular and humalog Somogyi effect -------- blood sugars continue to increase inspite of increasing insulin doses, caused by increased release of glucagon and cortisol Tx for somogyi effect -------- decrease amount of insulin dawn phenomenon -------- occurs related to the release of cortisol in the early morning causing high morning BS, need to check BS between 2-4 am

Tx of dawn phenomenon -------- higher insulin doses in evening or before bedtime oral hypoglycemic meds -------- used with type 2 DM, works on insulin so that it will work to unlock the cell, avoid during pregnancy first generation sulfonylureas -------- diabinaese, orinase, usually given before breakfast and supper, can cause nausea, anorexia and hypoglycemia second generation sulfonylureas -------- amayrl, glucatrol, diabeta, micronase, more powerful, used on higher BS, given once a day before breakfast, don't stop abruptly alpha glucosidase inhibitors -------- precose, often used with sulfonylureas, given 15 min before meals three times a day, causes diarreah and flatulence biguanide compounds -------- glucophage, can't take 48 hrs before an MRI or 48 hrs after, avoid alcohol, causes N/V diarrhea insulin enhancing agents -------- avandia, actos, given once daily in the am before breakfast, can cause liver damage, liver function tests done meglitinides -------- prandin, starlix, causes risk for infection, protect from infection exubera -------- inhaled insulin, 1 mg equals 3 units, given 10 min before meals, usually given four times a day ketones -------- develop when there is no glucose in the cell so uses fat as glucose, as fat breaks down it releases ketones in your blood hyperglycemia -------- occurs when calories exceed the insulin available, caused by eating too many CHO's, not enough insulin, stress and illness blood sugar over 200 hypoglycemia -------- BS below 60, insulin exceeds calories, caused by too much insulin, not eating, exercising during peak insulin time DKA (diabetic ketoacidosis) -------- caused from very high BS, causing metabolic acidosis, pH will be down HCO3 will be down, makes glucagon go up instead of down like it's supposed to increases BS, pt will be in ICU with an insulin drip S/S of DKA -------- fruity smelling breath and urine, tachycardia, increased BP, increased ketones hyperkalemia -------- high potassium levels caused by DKA because the potassium goes out of the cell and the sodium goes in, which can cause heart problems HHNK (hyperasmolar hyperglycemic nonketonic syndrome) -------- occurs with type 2 DM, high blood sugar , cause polyuria, glucosuria, dehydration, can result in coma and death if not resolved how does DM affect circulatory system? -------- increases risk for heart disease, strokes, heart attacks, peripheral vascular disease which leads to amputation, bad circulation

how does DM affect eyes? -------- blood vessels change, diabetic retinopathy, can cause blindness how does DM affect the kidneys? -------- renal failure blood vessels that go to the kidneys are ruined blocking blood from going to the kidneys, monitor BUN and creatinine, GFR how does DM affect the nerves? -------- neuropathy, unable to feel, if they can its tingling, burning feeling, caused by poor control of BS S/S of hyperglycemia -------- extreme thirst, frequent urination, dry skin, hunger, blurred vision, drowsiness, nausea S/S of hypoglycemia -------- shaking, fast heartbeat, sweating, anxious, dizziness, hunger, impaired vision, weakness, fatigue, irritability, headache DIABETES MELLITUS -------- by the low secretion and /or utilization of insulin. has been diagnosed in approximately 18.2 million people, or 6.3% of the population in US HYPOGLYCEMIA -------- is caused by excessive secretion of insulin. seem in type 1 diabetes DIABETES MELLITUS CHARACTERIZED BY -------- sweet urine, excessive urine production, thirst, excessive hunger, and in some cases weight loss HYPERGLYCEMIA -------- elevated level of glucose in the blood IMPAIRED FASTING GLUCOSE -------- a fasting blood glucose leve of >/= 100 -126 also known as prediabetes casual/random blood glucose (RBG) >200mg/dL plus classic symptoms a casual/random blood glucose (RBG) between 144-199mg/dL is diagnostic of prediabetes FASTING -------- defined as no kilocalorie intake for at least 8 hours 2 FORMS OF DIABETES -------- type 1 and type 2 TYPE 1 -------- aka insulin-dependent diabetes mellitus (IDDM) juevenile onset diabetes, can occur at any age but normally seen in childhood. these patients can not survive without daily doses of insulin because the pancreas doesnt produce sufficient insulin for glucose uptake. these patients are prone to ketoacidosis and hypoglycemia KETOACIDOSIS -------- hyperglycemia and excessive ketones. complex and lifethreatening condition demands emergency treatment. clinical manifestations dehydration acidosis, and electrolytem imbalances. ketones are acids FASTING BLOOD SUGAR -------- (FBS) performed routinely on most diabetics. in preparation the patient should be instructed not to eat or drink for 8 hours before test. water is the exception as t will not interfere with test results. ideally the test should be done after atleast 3 days of unrestricted diet

NORMAL FBS -------- <100 mg/dL , a finding of 126mg/dL is diagnostic of diabetes mellitus RENAL THRESHOLD -------- when blood glucose reaches 180-200mg/100mLthe kidneys begin to spill glucose into the urine TYPE 2 -------- aka non-insulin-dependent diabetes mellitus (NIDDM) persons with type 2 are not insulin dependent or prone to ketoacidosis. most are obese and weight reduction usually imprroves their ability to process glucose OTHER DISORDERS THAT CAN CAUSE DIABETES -------- pancreatitis, cystic fibrosis, surgical remove of pancreas, Cushing's disease or pharmacological doses of glucocoritcoids GESTATIONAL DIABETES -------- (GDM) is the term for glucose intolerance in pregnancy ANATOMY OF THE PANCREAS -------- gland that lies behind the stomach. has both endocrine and exocrin secretions. EXOCRINE FUNCTIONS OF THE PANCREAS -------- include the flow of enzymes into the intestine through ducts ENDOCRINE FUNCTIONS OF THE PANCREAS -------- secretions (hormones) flow directly into the bloodstream FUNCTIONS OF INSULIN -------- insulin is the only hormone that lowers blood glucose. a person normally secretes insulin in response to an elevated blood glucose level. insuline decreases blood glucose by accelerating its movement from the blood into the cells.in the cells it metabolizes to yield energy. may be stored as glycogen or converted to fat HOW MUCH GLYCOGEN CAN BODY STORE -------- normally 0.4 pounds of glycogen which is equal to 800 kilocalories INSULIN INFLUENCES -------- it influences metabolism of protein and fat. stimulates entry of amino acids into cells and enhances protein formation. it enhances fat storage in adipose tissue and indirectly inhibits the breakdown of fat for energy. IF BODY HAS AMPLE GLUCOSE -------- if body has ample glucose available for energy, protein and fat need not be broken dow to meet energy needs IF BODY DOESNT HAVE GLUCOSE AVAILABLE -------- if body doesnt have glucose available for energy, it will use dietary proteins or break down internal body protein stores to meet its immediate need for energy BLOOD GLUCOSE CURVE -------- a normal blood glucose rang is usually about 70-100 mg/dL CAUSES OF DIABETES -------- include genetic factors, lifestyle and viral infections

LIFESTYLE -------- a healthy lifestyle is particularly important for the prevention of diabetes in genetically susceptible clients. excessive body fat, inactivity and stress are risk factors for diabetes WEIGHT LOSS AND DIABETES -------- at least a dozen studies show that it reduces insulin resistance and increases peripheral glucose uptake INACTIVITY -------- is a lifestyle risk factor that predisposes one to diabetes VIRAL INFECTIONS -------- links have been noted between viral epidemics and the onset of diabetes AUTOIMMUNE DISEASES -------- the body cannot recognize its own ceels but rather treats them as foreign invaders. the event that provokes this process usually is viral infections GLYCOSURIA -------- glucose in the urine, a abnormally high amount. POLYURIA -------- excessive urine output, caused by glycosuria POLYDIPSIA -------- excessive thirst, caused by polyuria SIGNS AND SYMPTOMS -------- common agents are Staphylococcus aureus and Candida albicans involving the skin and mucous membranes poor wound healing is related to decreased circulation COMPLICATIONS -------- both acute and chronic complications occur with diabetes mellitus. acute require immediate care. chronic include diseases of eye, kidneys, heart, and nervous system. chronic are responsible for death rates DIABETES AND DEHYDRATION -------- without insulin glucose cannot be transfered across the cell membranes into the cells the body excretes the excess water, glucose, and electrolytes in urine ACIDOSIS -------- because the body can only use the glycerol portion of the triglyceride molecule for glucose, the fatty acid portion is processed into ketones. normally ketones are metabolized and excreted. in ketoacidosis the body overloads KETONURIA -------- ketones in urine KETONEMIA -------- ketones in blood KUSSMAUL RESPIRATIONS -------- caused by decreased level of carbonic acid in the blood by increasing the excretion of carbon dioxide through involuntary deep, gasping rapid breaths ELECTROLYTE IMBALANCES -------- regardless of the serum concentrations of potassium and sodium the pathological process of diabetic ketoacidosis depletes the electrolytes. either hypokalemia or hyperkalmia can lead to cardiac arrhythmias and must be carefully managed in ketoacidosis HYPOKALEMIA -------- low potassium

HYPERKALEMIA -------- high potassium 4 SIGNS OF HYPERGLYCEMIC HYPEROSMOLAR NONKETOTIC SYNDROM (HHNS) -------blood glucose level greater than 600mg/dL, absence of slight ketosis, plasma hyperosmolity, and profound dehydration. HHNS is like DKA except that the insulin deficiency is not as severe WHO IS SUSCEPTIBLE TO HYPERGLYCEMIC HYPEROSMOLAR NONKETOTIC SYNDROM (HHNS) -------- usually seen in elderly or people with undiagnosed type 2 diabetes. CAUSES OF HHNS -------- medications that increase blood glucose levels, chronic disease, and infections HYPOGLYCEMIA -------- caused by too much insulin (accidental or deliberate), too little food intake, a delayed meal, excessive medications such as oral hypoglycemic agents SYMPTOMS OF HYPOGLYCEMIA -------- confusion, headache, double vision, rapid heart rate, sweating, hunger, seizure and coma CHRONIC COMPLICATIONS OF DIABETES -------- diabetic retinopathy, lack of sensation in extremities, neuropathy, gastroparesis, nephropathy DIABETIC RETINOPATHY -------- a disorder that involves the retna. diabetes is the leading cause of blindness LEADING CAUSE OF BLINDNESS IN THE U.S. -------- diabetes DIABETIC NEUROTHAPY -------- chronic complication of diabetes mellitus, may complain of lack of sensation in extremities. may puncture or cut foot and not feel any pain NEUROPATHY -------- can affect gastric and intestinal motility, erectile function, bladder function, cardiac function and vascular tone GASTROPARESIS -------- paralysis of the stomach with delayed gatric emptying, may occur and alter the absorption of meal, which makes glycemic control problematic DIABETIC NEPHROPATHY -------- aka kidney disease TREATMENT FOR DIABETES RELATED ILLNESSES -------- to normalize the blood glucose thoughout the day and control blood pressure and blood lipids DIABETES AND BLOOD LIPIDS -------- > age 40 with total cholesterol greater than or equal to 135mg/dL, without overt cardiovascular disease, statin therapy to achieve an LDL reduction of 30-40% regardless of baseline LDL, levels is recommended. the primary goal is an LDL <100 mg/dL SELF-MONITORING OF BLOOD GLUCOSE -------- many individuals monitor their own blood glucose levels with a device called a blood glucose meter

PHYSICAL ACTIVITY AND DIABETES -------- exercise plays a key role in the management of diabetes individuals with diabetes should carry a source of glucose in case of a decrease in blood glucose, as exercise decreases blood glucose levels EXERCISE AND TYPE 1 DIABETES -------- involves some risk for individuals with type 1 diabetes because it changes insulin requirements in sometimes unpredictable ways more than 24 hours after exercise PREVENTION OF WIDE SWINGS IN BLOOD GLUCOSE -------- care should be taken not to overeat. too much food will cause levels to go up to high and subsequently too low. exercise is best done 60 to 90 min after meals when the blood glucose is highest MEDICATIONS AND DIABETES -------- 2 type are used: insulin (used by type 1) oral hypoglycemic agents (used by type 2 if a medication is required) INSULIN -------- most all insulin used in the US is human insulin, can not be taken orally because gastrointestinal enzymes would digest it before absorption. administered intravensously or subcutaneously HUMULIN -------- human insulin- produced few allergic reactions BOLUS DOSE -------- is short acting and designed to cover needs for one meal BASAL DOSE -------- longer acting and usually injected once or twice a day MEDICATION AND DIABETES -------- ideally medication is planned around the diet, not vise versa ORAL HYPOGLYCEMIC AGENTS -------- lower blooc glucose levels in type 2 diabetes. these drugs stimulate insuline release from the pancreatic beta cells, reduce glucose output from the liver, and increase the uptake in tissues MEDICAL NUTRITIONAL MANAGEMENT -------- diabetes is directly related to how the body uses food, thus nutrition is essential component to management promotion of normal nutrition and dietary modification to control blood glucose and lipid levels NUTRITIONAL GOALS -------- the goal of medical nutritional therapy is to educate the person with diabetes to make changes in food and exercise habits that lead to improved metaboilic control GOAL PRIORITY IN DIABETES -------- a high priority for the person taking i sulin is to facilitate consistency in the timeing fo the meals and snacks to prevent wide swings in blood glucose this requires coordination among exercise insulin and food intake. HIGH PRIORTY IN DIABETES -------- for the individual with type 2 is achieving glucose, blood pressure, and lipid goals. to achieve these goals, diet is a conerstone of treatment.

weight reduction for these clients usually improves short term glycemic levels and longterm metabolic control MEAL FREQUENCY -------- consistent timing and meal size assist in stabilization of blood glucose levels in type 1 diabetes. in general people with diabetes benefit from eating on a regular basis (every 4 to 5 hours) while awake SURVIVAL SKILLS -------- the american diabetes associations recommend that the newly diagnosed client initially learn THE EXCHANGE LISTS OF THE AMERICAN DIETETIC AND AMERICAN DIABETES ASSOCIATIONS -------- this approach to meal planning, teaches the learner about food composition. the exchange lists is used to calculate energy nutrient distribution CARBOHYDRATES AND MONOSATURATED FAT -------- dietary guildlines for americans recommends all people choose a variety of fiber-containing food such as whole grains, fruits, and vegetables because they provide vitamins, minerals, fiber, and other substances that optimize health PROTEIN -------- the need for protein in the diabetic population is the same as for the general population, if renal function is normal. excessive amounts of dietary protein should be avoided in people with diabetes, just as members of the general population should avoid it GLYCEMIC INDEX -------- of food attempts to classify foods according to their impact on blood glucose. the higher the glycemic index value, the higher the blood glucose would be expected to rise after ingestion of the food FAT -------- primary goal regarding dietary fat in clients with diabetes is to decrease saturated fat and cholesterol intake CARDIOVASCULAR DISEASE AND DIABETES -------- overall it is at least 2-3x's more common in patients with type 2 diabetes CARBOHYDRATE COUNTING -------- refers to a teaching tool that includes 3 progressive levels of difficulty, achievement and self care to be mastered by client WHY CARB COUNTING? -------- because cho is assumed to be the main factor affecting postprandial blood glucose elevation, priority is given to counting the total amount of carbohydrate consumed at one meal and/or snack as opposed to the source of the carbohydrate SPECIAL CONSIDERATIONS -------- persons with diabetes frequently ask questions about nutritional problems related to viatamin and mineral supplementation, alcohol, acute, illness, eating out, and delayed meals ALCOHOL AND DIABETES -------- the moderate use of alcohol does not adversely affect diabetes in the well controll client

ALCOHOL AND THE INSULIN USER -------- -limit to 1 drink for women and 2 drinks for men (1 defined as 12oz beer, 5 oz wine, or 1.5 oz distilled spirits -drink only with food to minimize hypoglycemia -do not cut back on food -if history of abuse and abstain -abstain during pregnancy NUTRITION DURING ACUTE ILLNESS EPISODES -------- acute illnes affects everyone including the person with diabetes. colds anf flu-like syptoms can be fatal for some people with diabetes unless precautions taken DEHYDRATION AND DIABETES -------- is more rapid when electrolytes and fluids are not replaced. vomiting diarrhea and fever all result in fluid loss during acute illness the individual should be instructed to monitor his or her blood glucose level ever 2-4 hrs until symptoms subside MEAL PLANNING AND ACUTE ILLNESS -------- (1) increase water intake, even for those who can eat regular diet, (2) eating smaller, more frequent feedings; and (3) eatin soft easily digested foods HYPOGLYCEMIA IN DIABETES MELLITUS -------- immediate treatment goal is for a blood glucose level of <60mg/dL is to increase blood glucose levels to within a normal level. take care not to overtreat THE 15 - 15 RULE -------- if the blood glucose level is less tha 60mg/dL, 15 grams of carbohydrate should be consumed. fifteen minutes later, he/she should measure the blood glucose a 2nd time 15 GRAMS OF CARBOHYDRATES IS EQUIVELENT TOO: -------- 2-3 glucos tabs/6-10 lifesavers/or 4-6 oz of juice TEACHING SELF CARE -------- the better educated the individual is about diabetes the greater the likelihood of his or her avoiding the acute and chronic complications of this disease RECOMMENDATION REGARDING HYPOGLYCEMIA -------- dietary management of reactive hypoglycemia consists of avoiding simple carbohydrates and sometimes taking small frequent feedings Brand of pioglitazone -------- Actos Brand of pioglitazone + metformin -------- ActosPlusMet Brand of rosiglitazone -------- Avandia Brand of metformin -------- Glucophage Brand of glyburide -------- Micronase, Diabeta Brand of micronized glyburide -------- Glynase

Brand of glyburide + metformin -------- Glucovance Brand glipizide -------- Glucotrol Brand of glimepiride -------- Amaryl Brand of repaglinide -------- Prandin Brand of nateglinide -------- Starlix Brand of acarbose -------- Precose Brand of miglitol -------- Glyset Brand of pramlintide -------- Symlin Brand of exanatide -------- Byetta Brand of sitagliptin -------- Januvia Brand of sitagliptin + metformin -------- Janumet Sulfonylurea with highest hypoglycemia -------- glyburide (20-30% of users) Sulfonylurea with least hypoglycemia -------- glimepiride (2-4% of users due to preserved glucagon action, glipizide XL also has less hypoglycemia) Sulfonylurea preferred in renal impairment -------- glipizide (glyburide accumulates if CrCl<30) MOA of meglitinides (Prandin/Starlix) -------- Stimulates Insulin secretion (administer 1530 m before meals to dec PPG) Dose of Starlix -------- 120 mg TID ac Dose of Prandin -------- 0.5 - 4 mg TID ac MOA of metformin -------- decreases hepatic glucose and incease peripheral glucose uptake Advantages of metformin -------- no hypoglycemia, weight loss, decreased TG Disadvantages of metformin -------- nausea, decreased B12, lactic acidosis, (CI in SeCr > 1.5, hepatic impairment, and while on contrast) MOA of acarbose -------- alpha glucosidase inhibitor in brush-border Disadvantages of acarbose -------- abdominal pain, flatulence, diarrhea (start at low dose 25 mg QD and increase q2w, CI in IBDS and bowel obstruction)

MOA of thiazolidinediones (glitazones) -------- PPAR-gamma agonists (decrease hepatic glucose production and increase peripheral glucose uptake) AE of glitazones -------- hepatotoxicity (don't start if ALT 2.5 X ULN and stop if >3 X ULN), edema, anemia, (pio may decrease estrogens) Dose of pioglitazone -------- 15-45 mg qd Brand of rapid insulin Lispro -------- Humalog Brand of rapid insulin Aspart -------- Novolog Brand of regular insulin -------- Humalin, Novalin Brand of Intermediate isophane insulin -------- NPH Brand of long acting Glargine -------- Lantus Brand of long acting Detemir -------- Levimir Use of pramlintide -------- adjunct to prandial insulin not achieving PPG Dose of pramlintide -------- (Reduce short acting insulin 50% give) 60-120 mcg Use of exenatide -------- adjunct to improve glucosein DM2 on metformin/sulfonylurea (CI in CrCl <30) Dose of exenatide -------- 5-10 mcg BID (injection within 60 min of meal) Use of sitagliptin -------- monotherapy or with metformin or thiazolidinediones (less hypoglycemia than SU with same glucose lowering but must be renally adjusted) MOA of sitagliptin -------- DPP4 inhibitors (that increase incretin hormone )

Home

Unit X

Chapter 73

Student Home

Key Concept Summary

Key Concept Summary


1. The physiology of the respiratory system involves two main processes: perfusion and ventilation. Perfusion is the blood flow through the lungs, which allows for gas exchange across the capillaries. Ventilation is the process of moving air into and out of the lungs. The airway diameter is regulated by the autonomic nervous system, which can cause the airway to dilate or constrict. 2. Asthma is a chronic disease that has both inflammatory and bronchospasm components. The inflammatory component of asthma involves an increase in airway edema coupled with increased mucus secretions that contribute to airway obstruction. Bronchospasm may be induced by various triggers. Acute dyspnea and wheezing are common signs of asthma. Drugs are used to prevent asthmatic attacks and to terminate an attack in progress. 3. Inhalation is a common route of administration for pulmonary drugs because it delivers drugs directly to the sites of action. The inhalation route is used to deliver medications directly and safely to the respiratory system. Aerosol medications are those delivered as very small liquid droplets or fine, dry particles. Nebulizers, MDIs, and DPIs are types of devices used for aerosol therapies. 4. The goals of asthma pharmacotherapy are to terminate acute bronchospasms and to reduce the frequency of asthma attacks. National Asthma Education and Prevention (NAEPP) guidelines are used in asthma management. These guidelines initiate therapy in a stepwise approach based on the severity of asthma symptoms. The goals of asthma therapy are to terminate acute bronchospasms and to prevent asthma attacks. Medications used in asthma management are classified as quick-relief agents or long-term control agents. 5. Beta2-adrenergic agonists are the most effective drugs for relieving acute bronchospasm. Beta agonists activate beta2 receptors in bronchial smooth muscle to cause bronchodilation. The shortacting beta agonists have a rapid onset of action and are used to terminate acute bronchospasm. The long-acting beta agonists are prescribed for asthma prophylaxis, usually when corticosteroids fail to achieve symptom control. 6. The inhaled anticholinergics are used for preventing bronchospasm. Ipratropium and tiotropium act by blocking cholinergic receptors in bronchial smooth muscle. Ipratropium is used as an alternative drug for asthma prophylaxis and intranasally as a decongestant. Tiotropium is used to prevent bronchospasm in patients with chronic bronchitis or emphysema. 7. Inhaled corticosteroids are the most effective drugs for the long-term control of asthma. Corticosteroids are the most potent natural anti-inflammatory substances known. Inhaled corticosteroids are the drugs of choice for the prevention of asthmatic attacks and the management of chronic asthma. Oral corticosteroids may be used for the short-term management of acute asthma exacerbations. 8. Mast cell stabilizers are used for the prophylaxis of asthma and act by preventing the release of histamine.

Mast cells contain inflammatory granules, such as histamine, that mediate inflammatory and allergic reactions. When these cells are sensitized, they release the inflammatory substances into the body where they initiate an inflammatory response. Mast cell stabilizers are considered alternate drugs for the prophylaxis of mild to moderate asthma symptoms. 9. The leukotriene modifiers, which are primarily used for asthma prophylaxis, act by reducing the inflammatory component of asthma. Leukotriene modifiers are medications that reduce inflammation and are considered alternate drugs in the prophylaxis of persistent asthma. Zileuton acts by blocking lipoxygenase, the enzyme that controls leukotriene synthesis. Montelukast and zafirlukast block leukotriene receptors. They are not considered bronchodilators, although they do reduce bronchoconstriction indirectly. 10. Methylxanthines were once the mainstay of asthma pharmacotherapy but are now rarely prescribed for that disorder. Methylxanthines such as theophylline are less effective, have a narrow therapeutic index, and produce more adverse effects than the beta agonists. They are primarily reserved for the long-term management of persistent asthma that is unresponsive to beta agonists or inhaled corticosteroids. 11. Monoclonal antibodies are a newer form of therapy for the prevention of asthma symptoms. Omalizumab is the only biologic therapy for asthma management. The drug binds to IgE, preventing the release of chemical mediators of inflammation. It is used for treating moderate to severe, persistent asthma that cannot be controlled with inhaled corticosteroids. 12. Chronic obstructive pulmonary disease (COPD) may be treated with bronchodilators, anti-inflammatory agents, and mucolytics. COPD is a progressive disorder characterized by chronic and recurrent obstruction of airflow. The two most common conditions that cause chronic pulmonary obstruction are chronic bronchitis and emphysema. The goals of the pharmacotherapy of COPD are to relieve symptoms and avoid complications of the condition. Multiple pulmonary drugs such as bronchodilators, anti-inflammatory agents, expectorants, mucolytics, antibiotics, and oxygen may offer symptomatic relief.

13.
14.
aerosol asthma

Chapter 73 Key Terms


suspension of minute liquid droplets or fine solid particles in a gas chronic inflammatory disease of the lungs characterized by airway obstruction rapid constriction of the airways recurrent disease of the lungs characterized by excess mucus production, inflammation, and coughing

bronchospasm chronic bronchitis

generic term used to describe several pulmonary conditions chronic obstructive characterized by cough, mucus production, and impaired gas pulmonary disease exchange (COPD) device used to convert a solid drug to a fine powder for the purpose dry powder inhaler of inhalation (DPI) emphysema leukotrienes terminal lung disease characterized by permanent dilation of the alveoli chemical mediators of inflammation stored and released by mast

cells; effects are similar to those of histamine metered-dose inhaler (MDI) methylxanthines mucolytics nebulizer perfusion small volume nebulizer device used to deliver a precise amount of drug to the respiratory system chemical class for theophylline and caffeine drugs used to loosen thick mucus device used to convert liquid drugs into a fine mist for the purpose of inhalation; also called small volume nebulizer blood flow through a tissue or organ device used to convert liquid drugs into a fine mist for the purpose of inhalation; also called nebulizer

condition characterized by repeated seizures or one prolonged status asthmaticus seizure attack that continues for at least 30 minutes ventilation process by which air is moved into and out of the lungs

Your Results for "NCLEX-RN Review"

Print this page

Student results on this activity were sent to the Grade Tracker. Note that it may take a few moments for the score to appear. Go to Grade Tracker. Summary of Results for Litta Oglesby Site Title: MyNursingKit for Pharmacology: Connections to Nursing Practice Book Title: Pharmacology: Connections to Nursing Practice Book Author: Adams Location on Site: Unit X > Chapter 73 > Student Home > NCLEXRN Review Submitted: March 11, 2012 at 11:47 PM (UTC/GMT) 3 correct: 17% 17% Correct of 10 questions 15 incorrect: 83%

2 questions contain multiple pairs, scored for a total of 10 questions. More information about scoring 1. A client with asthma asks which of the prescribed medications should be used in the event of an acute episode of bronchospasm. The nurse will instruct the client to use: Your Answer: Beclomethasone, a glucocorticoid anti-inflammatory drug, by inhalation. Correct Answer: Albuterol, a beta agonist bronchodilator, by inhalation. Rationale: There are two important items to consider: (1) the medication and (2) the route.

A drug to abort bronchospasm should be given by inhalation in order to ensure rapid action directly at the site. An inhaled beta agonist such as albuterol meets both criteria. Option 2 is incorrect because although inhalants are delivered directly, they do not work quickly and are used for prevention of inflammation. Option 3 is incorrect because this anticholinergic is not approved as rescue therapy for treatment of acute bronchospasm. Option 4 is incorrect because leukotriene modifiers are indicated for prevention of respiratory problems, not for treatment of acute bronchospasm. Furthermore, giving a medication PO would not be appropriate when treating acute bronchospasm. Cognitive Level: Analysis; Client Need: Safe, Effective Care Management; Nursing Process: Implementation

2.

A client is prescribed beclomethasone (Beclovent), a glucocorticoid inhaler. Education by the nurse will include: Your Answer: Check your heart rate because this may cause tachycardia. Correct Answer: Rinse your mouth out well after each use. Rationale: Glucocorticoids can decrease the beneficial oral flora that will allow for an overgrowth of fungal infections such as candida. Rinsing the mouth removes any glucocorticoid drug deposited there, and prevents it from being being swallowed. Thus it decreases the likelihood of toxicity through systemic absorption. Option 1 is incorrect because it is the bronchodilators (e.g., adrenergic agonists, anticholinergics, and xanthines) that are likely to cause tachycardia, not glucocorticoids. Option 2 is incorrect because it is the xanthines (e.g., aminophylline and theophylline) that are chemically related to caffeine, not the glucocorticoids. It would not be restricted with a glucocorticoid. Option 4 is incorrect because it is the bronchodilators (e.g., adrenergic agonists, anticholinergics, and xanthines) that are likely to cause the client to feel shaky and nervous. Cognitive Level: Application; Client Need: Safe, Effective Care Management; Nursing Process: Implementation

3.

The nurse should inform the client who is prescribed a nebulizer treatment with a bronchodilator agent that a common adverse effect is: Your Answer: Temporary dyspnea. Correct Answer: An increased heart rate with palpitations. Rationale: Bronchodilators (e.g., beta agonists, anticholinergics, xanthine derivatives) have an adverse effect on heart rate elevation and palpitations. Option 2 is incorrect because bronchodilators do not decrease the immune response the way certain antiinflammatory agents do. Option 3 is incorrect because bronchodilator increase alertness. Option 4 is incorrect because bronchodilators relieve dyspnea. While some bronchodilators have been known to cause unexpected problems and paradoxical bronchospasm, this is uncommon and the question asks for a common adverse effect.

Cognitive Level: Comprehension; Client Need: Safe, Effective Care Management; Nursing Process: Implementation

4.

The nurse should monitor the client who is taking corticosteroids for evidence of: Select all that apply. Your Answers: Hyperglycemia. Urinary retention. Correct Answers: Infection. Hyperglycemia. Correct. Rationale: Because corticosteroids decrease the immune response, the likelihood of infections is increased. Glucocortoids also increase blood glucose thus increasing the likelihood of hyperglycemia. Corticosteroids do not cause urinary retention (option 3) and do not increase the likelihood of tachycardia (option 4). Corticosteroids do not cause photophobia (option 5). Cognitive Level: Application; Client Need: Health Promotion and Maintenance; Nursing Process: Assessment

5.

A 4-year-old child with respiratory distress secondary to asthma has an order for a nebulizer treatment. The type of medication most likely to be given for asthma management is a: Your Answer: Corticosteroid. Correct Answer: Beta agonist. Rationale: Beta agonists are agents that are used in the management of asthma that may be given to children younger than 5, and are available in formulations suitable for nebulizer treatments. The agents in options 2, 3, and 4 do not meet one or more of the criteria listed above. Cognitive Level: Analysis; Client Need: Health Promotion and Maintenance; Nursing Process: Planning

6.

Despite repeated demonstrations of proper inhaler use by the nurse, the client is unable to return a proper demonstration on the training inhaler. The client is becoming frustrated. The best action for the nurse to take is to: Your Answer: Provide a spacer for use with the inhaler.

Rationale: Some clients have difficulty mastering the coordination between inhalation and activation of the medication. In these instances, a spacer will hold the medication cloud so that this is not a concern. The spacer has additional advantages because it results in a more effective delivery of the drug to the site of action and less drug deposition in the mouth and oropharynx. Additional practice may help in the long term, but it is not the priority for an immediate solution to the problem (option 1). The health care provider would not need to be contacted because the client has difficulty learning, provided that a solution is readily available (option 2). Substitution of an oral form of drug is not in the nursing scope of practice and, even if it were, an oral formulation would not be a suitable substitute because the onset of action would be delayed (option 4). Cognitive Level: Analysis; Client Need: Health Promotion and Maintenance; Nursing Process: Planning

7.

A 60-year-old man is prescribed ipratropium (Atrovent) for the treatment of asthma. The appropriate nursing intervention includes: Your Answer: Teaching the client to report the inability to urinate. Rationale: Ipratropium in an anticholinergic agent. Anticholinergics can cause urinary retention. Although urinary retention is uncommon with inhalant medications, clients should be aware of this potential side effect. Caffeine is not contraindicated for clients taking anticholinergic agents (option 1). Anticholinergic agents do not cause problems resulting in liver enlargement (option 2). These agents are more likely to cause constipation, not diarrhea (option 4). Cognitive Level: Application Client Need: Health Promotion and Maintenance Nursing Process: Implementation

8.

Match each prototype on the left to the category of drug it represents. Option Your Answer Correct Answer

8.1

Beta-adrenergic agonists Anticholinergic corticosteroids Corticosteroids

B. beclomethasone (Beclovent, D. albuterol (Proventil, Beconase, Vancenase, Vanceril) Ventolin, Volmax) A. zafirlukast (Accolate) D. albuterol (Proventil, Ventolin, Volmax) C. ipratropium (Atrovent) C. ipratropium (Atrovent) B. beclomethasone (Beclovent, Beconase, Vancenase, Vanceril) A. zafirlukast (Accolate)

8.2 8.3

8.4

Leukotriene modifiers

Rationale: Each prototype represents a specific category of agent. Cognitive Level: Knowledge Client Need: Health Promotion and Maintenance Nursing Process: Implementation

9.

A client who is prescribed 400 mg/day of theophylline smokes two packs of cigarettes per day. The nurse knows that this will pose what complication? Your Answer: There will be an increased risk of theophylline toxicity. Correct Answer: The dose may be inadequate to manage symptoms. Rationale: Smoking increases the clearance of the theophylline; therefore, a larger than usual dose may be required to maintain a therapeutic level of medication. Otherwise, the dosage is one that is adequate and safe. Smoking does not increase the stimulant effect (option 1). Smoking would not contribute to the likelihood of theophylline toxicity (option 3) or systemic side effects (option 4). Cognitive Level: Knowledge Client Need: Health Promotion and Maintenance Nursing Process: Planning

10 .

Match each category of drug with its primary effect as a bronchodilator or antiinflammatory agent. Option Your Answer A. Anti-Inflammatory Agent B. Bronchodilator D. Anti-inflammatory agent E. Anti-inflammatory agent C. Bronchodilator F. Bronchodilator Correct Answer A. Anti-Inflammatory Agent F. Bronchodilator E. Anti-inflammatory agent C. Bronchodilator D. Anti-inflammatory agent B. Bronchodilator

10.1 10.2 10.3 10.4 10.5 10.6

Corticosteroids Methylxanthines Mast cell stabilizers Anticholinergic corticosteroids Leukotriene modifiers Beta-adrenergic agonists

Rationale: Each category has a specific effect that plays a role in management of pulmonary disorders. Cognitive Level: Knowledge Client Need: Health Promotion and Maintenance Nursing Process: Implementation

You might also like